Download as pdf or txt
Download as pdf or txt
You are on page 1of 120

pregnancy

1 Which hormone is associated with fetal productive abnormalities?


Androgen, Estrogen, progesterone

2 Doxycyclin in pregnancy?
Category D

3 Rifampin in pregnancy?
category C

4 Antibiotic in pregnancy?
Penicillins, including amoxicillin, ampicillin.
Cephalosporins, including cefaclor, cephalexin.
Erythromycin.
Clindamycin

5 Vaccines in pregnancy?
Tdap vaccine and flu shot (inactivated live influenza)

6 Calcium dose in pregnancy?


(1.5 g–2.0 g oral elemental calcium) is recommended for pregnant women to reduce the risk of
pre_eclampsia.

7 Used as galactagogic increase prolactin


Domperidone

8 HIV therapy used in pregnancy??

9 Finasteride when used during pregnancy


Induced abortion
Spontaneous abortion
Endometrial hypertrophy

10 Pregnant woman in hospital for delivery dr want to induce uterine contractions so he gives
her intravaginal tablets of ?
Dinoprostone

11 Pregnant woman has candida albicans so ttt ?


Clotrimazole
12 Which one of these more liable for influenza complications ?
Pregnant woman and postpartum

13 The label which must be written for vincristine vial?


Fatal if given intrathecal only iv infusion

14 Contraceptive in postpartum and lactating woman?


Progestin

15 Which one associated with groth restriction in pregnancy


Lisinopril

16 What is the meaning of category c?


Animal studies show minor risk and human studies are not available Or neither Animal
studies nor human studies done

17 When to start folic a to prevent spina bifida?


Before 1 months

18 Drugs that increase & decrease prolactin level?


Increase prolactin level
(dopamine antagonist)
Antipsychotics second generations
Decrease prolactin level
(dopamine agonist)
Bromocriptine
Cabergoline

19 HIV medication that given during labor


Zidovudine 2 mg/kg by infusion over 1 hr

20 Treatment of glaucoma in pregnancy


Latanoprost

21 Vitamin should be avoided in high doses in case of pregnancy?


Vitamin A

22 Laxative should be avoided in pregnancy?


Lubricants, mineral oils ( castor oil)

23 Drug in pregnancy for hypertension


labetalol, nifedipine extended-release and methyldopa

24 Finasteride what cause to pregnancy if use ?


Category X

25 Which drug should pregnant start before pregnancy 1 month?

26 Pregnant has pain in 3rd trimester what should she take?


Acetaminophen
27 Drug must be stopped one month prior pregnancy
Isotretonin
28 Drug contraindication in pregnancy in 2-3 trimester?

29 which drug is safe in pregnant


Ampicillin

30 Pregnant with DVT ?


Use enoxaparin

31 Drug product prolactin ?


Drugs that act as dopamine antagonists (for example, metoclopramide and antipsychotics
such as risperidone) can increase the secretion of prolactin.] Second generation of
antipsychotics also increase prolactin levels

32 A pregnant women with high blood pressure she take labetalol but blood pressure remain
high what should we add
Mgso4

33 A women suffering from facial flushing and vaginal dry what is the best drug for her
Estrogen

34 Pregnant with headache


Paracetamol

35 A women has uterus cancer she refused chemotherapy because it cause hair fall she also
refused to take out her uterus because she want to get pregnant
Explain the risk and benefit and let her decide

36 Drug used for treatment of premature ejaculation


Dapoxetine
37 Which antipsychotic drug is not given to pregnant
Bupropion

38 woman take levothyroxine 50 and she became pregnant and she feel fatigue
double the dose 1s trimester

39 Drugs if used in the first trimester in pregnancy cause anemia of the fetus
Ciprofloxacin

40 effect of nicotine in pregnant ?


Decrease blood flow to uterine

41 Why we don’t use of NSAID in trimester


NSAIDs should generally be avoided due to the risk of premature closure of the ductus arteriosus

42 When take Warfarin make teratogenic in

43 What mean category C


Animal reproduction studies have shown an adverse effect on the fetus and there are no
adequate and well-controlled studies in humans, but potential benefits may warrant use of
the drug in pregnant women despite potential risks.

44 Which drug contraindicated in pregnancy ?

45 Pregnant women with schizophrenia

46 Eclampsia situation
Eclampsia is a severe complication of preeclampsia. It’s a rare but serious condition where
high blood pressure results in seizures during pregnancy.

47 The medication is approved for increase prolactin secretion?


dopamie antagonist

48 The vaccination is contraindicated in pregnancy?


HPV, MMR, BCG, LAIV, Varicella and ZVL

49 Women with hypothyroid then Pregnant what do to levothyroxine


Increase the dose 30-50%

50 Treatment nausea and vomiting in pregnant woman


Vitamin B6

51 Treatment of glaucoma in pregnancy


Brimonidin

52 responsible for milk production


Lactobacillus bacteria

53 Drug inhibit Estrogen synthesis


Tamoxifen
54 Drug reduce prolactin
Bromocriptine

55 What dilate cervix during labor


Oxytocin

56 A 37 years smoker female want contraceptive what is the best for her
Progestin

57 what is the effect of smoking in pregnancy


reduce blood flow to uterine not sure

58 drug cause hyperprolactinemia


Metoclopramide

59 oral contraceptive work on ?


Gnrh hormone in phase of ovulation

60 hydralazine is ?
category C pregnancy

61 paracetamol is ?
category B for pregnancy

62 bromocriptine ?
reduce prolactin level

63 for female with DVT , combination contraceptive shouldn't be used birth pills) ?
use only progesterone pills(levonorgestrel)

64 Combination contraceptive act on ?


Ovulation

65 which of the following is more prone to influenza symptoms and problems ?


Pregnant

66 Captopril is contraindicated with ?


Pregnancy

67 Which of the following used as anti estrogen when used cause abortion ?
Anastrozole

68 Pregnant woman taking valporic acid go to physician with tonic- clonic seizures. Which of the
following is true?
use valporic with folic acid
go to the doctor

69 Pregnant woman has UTI (Urinary Tract Infection) which is the drug of choice to treat her UTI ?
Nitrofurantoin
70 Pregnant woman with G6PD deficiency has G-ve. M.o. and UTI, which is the drug of choice to treat
her UTI ?
Cefuroxime

71 Which one of the following is taken orally ?


Estrogen gluconate

72 about sid effect of oral contraceptive except ?


Constipation

73 a pregnant woman at 43 week pregnancy She began labor actions for 12 hrs. The contractions were
strong at first but after some while became very weak Which of the following could be useful in her
case ?
Oxytocin

74 A pregnant women in her 35 week has a headache she should take ?


acetaminophen(Paracetamol)

75 Paracetamol category b in pregnancy means ?


Used with caution

76 A pregnant women with syphilis should take ?


Amoxicillin

77 Which the following hormone suppress due to take oral contraception ?


GNRH (Gonadotropin-releasing )

78 MOA of Chorionic gonadotrophic during of treatment of infertility?


Follice and induce ovulation

79 A pregnant woman has deep vein thrombosis (DVT) hospitalized treated by ?


set unfractionated heparin

80 which of the following prevents Neural tube defects ( birth defects ) ?


Folic acid ( vit . B9 )

81 Category X ?
Category X Studies in animals or humans have demonstrated fetal abnormalities and / or there is
positive evidence of human fetal risk based on adverse reaction data from investigational or
marketing experience , and the risks involved in use of the drug in pregnant women clearly
outweigh potential benefits . Example drugs : atorvastatin , simvastatin , warfarin , methotrexate ,
finasteride

82 pregnant hypertensive woman take ?


Methyldopa

83 Dose of folic acid to ?


non pregnant : 400 mcg / day
pregnant : 600 mcg / day
Lactating : 500 mcg / day
84 a postmenopausal old woman suffering from facial flushing and vaginal drying She has done
hysterectomy procedure . which drug of the following should she use ?
estrogen

85 drugs act as antiestrogen


Clomiphene, tamoxifen., raloxifen, anastrazol, letrozol

86 Pregnant woman with G6PD deficiency has G-ve. M.o. and UTI, which is the drug of choice to treat
her UTI:
Cefuroxime

87 A pregnant women senstive to amoxicillin which is the Drug Of Choice for her disease ... what will
be the suitable altrenative anti-biotic for her ?
Erythromycin

88 misoprostol is .............................
category X

89 A 39 years old what is oral birth control pills appropriate for her
Ethinyl estradiol / lnestrenol

90 for female with DVT, which is contraindicated ...


Combined contraceptives
(Estrogen containing contraceptive)

91 A women takes estrogen and she made hysterectomy should take?


Hydroxyprogestrone

92 antiestrogen cause abortion ..


Anastrozole

93 Antiprogesterone cause abortion.


Mifepristone

94 Antiprogestin causes abortion


is: Mifepristone

95 Pregnant with HTN?


Labetalol

96 HIV medication for pregnant ?


Zidovudine

97 All of these Vaccine are contraindicated in pregnancy except?

98 calcium dose with pregnant ?


1200
Anesthetic

1 Paracetamol dose in emergency department where can you find it?


ER dose 650mg

2 Drug worsen case of paracetamol overdose?


Ethanol

3 Hepatotoxicity of paracetamol due to ?


Glutathione depletion

4 patient used morphine 60mg twice daily he want to change fentanyl patch
50mcg

5 morphine extra dose to decrease pain


10mg

6 constipation
SE/ morphine

7 Skeletal muscle relaxant


Baclofen

8 Frist approach to diaper inflammation


Petrolatum

9 Which of these IV anesthesia need hepatic adjustment or CI in hepatic failure

10 Which local anesthesia cause more hepatotoxicity

11 Morphine convert to codeine


by dealkaylation

12 analgesic for acute coronary syndrome and fibrinolytic for stroke


Gentamicin trough peak monitoring

13 morphine cause ?
Constipation

14 Drug that increases paracetamol toxicity ?


Alcohol (ethanol)

15 Example of pathological toxicity ?


Hepatic necrosis from acetaminophine

16 Morphine causes respiratory depression by ?


depression of respiratory center in brain

17 most potent anti-inflammatory corticosteroid systemically is ?


Dexmethasone

18 Enkephalins are peptides that ?


similar in action to morphine
19 acetyl salicylic acid used in ?
Analgesic

20 Opioids can be used as ?


Analgesics

21 which drug may used as anti-spasmodic ?


Baclofen

22 To increase duration of lidocaine in anesthesia ?


Epinephrine

23 which of the following anesthetics cause cardiotoxicity ?


b-Bupivcaine

24 local anesthesia agent is ?


Lidocaine

25 patient with hepatic and renal failure which of the following skeletal muscle relaxant can be used ?
Atracurium

26 with anti-inflammatory and analgesic ?


Ibuprofen

27 the recommended maximum adult daily dose of paracetamol


4000 mg

28 maximum dose of ibuprofen ?


1200

29 codeine loss its activity for pain relief with


poor metabolizer
Angina

1 Nitroglycerin MOA
Nitroglycerin, an organic nitrate, is a vasodilating agent that relieves tension on vascular
smooth muscle and dilates peripheral veins and arteries. It increases guanosine 3'5'
monophosphate (cyclic GMP) in smooth muscle and other tissues by stimulating guanylate cyclase
through formation of free radical nitric oxide.

2 Treatment of acute angina attack?


Nitroglycerin sublingual

3 Patient with angina which one can worse his case?


Celecoxib

4 Septic shock first line?


Iv fluids

5 Patient with angina worse with


Celecoxib

6 Drug of choice in prinzmetal angina ?


Nitroglycerin and calcium channel blockers

7 Septic shock first management


Norepinephrine

8 which of the following used in treatment of anaphylactic shock ?


epinephrine ( used as broncho dilator)

9 Why Beta blockers are used to treat angina ?


decrease sympathetic cardiac stimulation

10 which one is not effect of Nitroglycerin vasodilation ?


slow heart rate

11 beta blocker used in angina because .


decrease sympathomimtic cardiac stimulation
CKD

Most popular to cause acute kidney injury


NSAID

2 What is the best way to protect the kidney from acute renal failure (ARF
Adequate hydration

3 Renal adjustment needed


All need adjustment sitagliptin- voriconazole- fluconazole- Insulin glargine

4 cushing - like syndrome is due to ?


adrenal hyperplasia

5 Kidney failure patient with hyperkalemia which of the following should be used ?
calcium carbonate (may use calcium gluconate or calcium carbonate)

6 The most dangerous adverse effects of Amikacin is ?


(kidney disease)

7 Hb1genotybe hepatic patient (hepatitis C genotype 1) fot ttt take ?


Interferon and ribavirin

8 Drug cause renal disease ?


Aminoglycosides

9 Patient has Low creatinine clearance= high serum creatinine=kidney injury contraindicated in this
case is?
Spironolactone

10 Hb1V genotype hepatic patient for ttt take ?


interferon and ribavirin

11 vitamin d for patients with renal failure ?


1,25 - dihydroxycholecalciferol Calcitriol )

12 A patient with renal failure on erythropoietin therapy. which of the following may decrease the
effect of therapy ?
iron deficiency

13 The importance of ceriatinine in monitoring kidney disease:


bound to protein and excreted by kidney

14 Mg antacid for patient with renal failure the antacid will cause
decrease the Elimination Of Mg

15 Drug known to cause kidney injury


Aminoglycoside

16 to avoid acute renal failure


maintain hydration

17 Which of the following Modifications risk factor in acute renal failure ?


HTN
Bacteria

1 Salmonella make diarrhea 10 in 10,000 people and shigella makes diarrhea 100 in 10,000
people? Choose the correct statement
shigella is more pathogenic than salmonella

2 Pseudomonas shape?
Rod shape bacteria

3 Actinomyces shape?
Rod shape bacteria

4 Endotoxins characteristic
heat stable

5 Microorganism responsible for dental carries


Streptococcus mutans

6 Bacterial genome consist of ?


single strand DNA

7 What mean facultative anaerobic?


Aerobic but can live in absence of oxygen

8 Bacteria responsible for food poisoning


Staphylococcus aureus

9 Differentiate between gram +ve & -ve bacteria


Gram- positive organisms have a thick cell wall and stain dark purple, or bluish in color
from the crystal violet stain.
Gram-negative organisms have a thin cell wall and take up the safranin counterstain,
Resulting in a pink or reddish color
10 Clostridum and psudonomus which related
Difficale

11 Diphtheria is an infectious disease caused by the bacterial microorganism known as


Corynebacterium

12 Which bacteria take cluster shape


Staphylococcus

13 Patient take 75mg vancomycin every 12 hours and the level is 7mch/dl the range level is
(20-10) the patient need dose adjustment to reach 10mcg/dl what is the dose needed
82mg – 105mg

14 Drug has role in RSV ( respiratory syncytial virus ) in newborn ?


Palivizumab for prevention and treatmen

15 Bacteria causes osteomyelitis


Staphylococcus aureus

16 Bacteria cause plague


Yersinia pastis
17 Obligate anaerobic M.O
Bacteroides fragilis

18 Ebola transmission
Blood or body fluids

19 probiotics are ?
bacteria and yeast eaten to provide patient with health

20 Release from bacteria during The growth ?


Exotoxin

21 Which organism is the cause of travelers’ diarrhea ?


E.coli

22 Which of the following is released by bacterial cell wall during phagocytosis ?


Endotoxin

23 probiotics are ....


live yeast or bacteria

24 How to differentiate bet p.argeunosa & E.coli?


Lactose fermentation

25 What is the Essential element of Fungai wall?


Chitin

26 makeup of cell wall fungi?


Ergostrol
Osteoporosis
1 What is the most common side effect of bisphosphonate
irritation of the upper gastrointestinal (GI) mucosa

2 Which bisphosphonate is associated with gastric ulcer?


(Alendronate, Risedronate)

3 Which treatment of osteoporosis could be given monthly?


Ibandronate given monthy

4 Which one of osteoprosis drugs need renal adjustment before use


Zoledronic acid – teriparatide

5 Therapeutic effect of glucosamine and chondroitin is used for?


Osteoarthritis

6 Patient with osteoarthritis suffering from pain


Acetaminophen

7 Prevent bone fracture in osteoporosis


Ibandronate

8 zoledronic acid monitoring?


Renal

9 Side effects Bisphosphonate


Hypocalcium ,

10 potential side effect of Bisphosphonates


osteonecrosis of jaw

11 Drug induce esophageal ulcer


Alendronate

12 etidronate used for ?


osteoporosis or paget disease

13 an elderly women with osteoporosis after giving her Calcium + vitamin D she become better , so we
give her in the next step ?
alendronate

14 geriatric patient with osteoarthritis mild pain in hip . he has hypertension and coronary artery
disease what is the best drug for pain ?
paracetamol(acetaminophen)

15 drug interaction between ?


furosemide and osteoporosis (cause hypocalcemia)

16 In the treatment of the osteoporosis, which of the following is not correct ?


hormone replacement therapy (HRT) should be considered in all
17 a patient come to you taking alendronate, you advise him ?
take alendronate 1/2 hour before breakfast with water and stand upright for 1/2 hour

18 for a patient with osteoarthritis suffering from pain ?


Acetaminophen

19 Dose of calcium for woman 65year ?


1200mg

20 old man have rheumatoid arthritis taking (methotrexate - ibuprofen - -


Losec) and these drugs were not effective, so the next step we use:
Lefulonamide

21 a patient come to you taking alendronate, you advise him


take alendronate 1/2 hour before breakfast with water and stand
upright for 1/2 hour

22 What is the osteoporosis medication that approved for postmenopausal women to be taken
monthly?
Bisphosphonates- ibandronate

23 What is the Patient education regarding bisphosphonate?


Take it before breakfast for 30min with full cup of water and maintain an upright position for 30min
( 60 min for ibandronate)

24 What is the patient counseling with alendronate ?


Patients are advised to not lie down for at least 30 minutes after taking alendronate and Sit upright
or stand upright until at least 30 minutes

25 Used to Diagnose osteoporosis ?


DEXA, or DXA

26 Daily dose of Ferrous Sulfate??


300 mg bid or TID

27 Iron storage in the body ?


Hemosiderin

28 calcium dose with pregnant ?


1200

29 with osteoporosis, drug prevent hip fracture ?


Alendronate
Communication

1 Patient has green yellow phlegm


Referral to physician

2 Patient take two eye drop in same time should separate?


Separate five minutes

3 Patient receive prescription 17 December 2018, has 5 refill , came back in April 2019 asking
for refill
Have one more refill

4 Example of intentional non-adherence ?


Patient does not take medicines because of fear of side effect

5 Which one is prescribing error?


Wrong drug order

6 Which one is of 5 rights of drug administration?

7 Which one is the last person to prevent medication error?

8 Nonverbal communication which one?


Tone of voice Rate and volume of speech Facial expression Eye contact Gestures/touch

9 Patient information how to deal with between practitioner


do not share it with anyone

10 Look alike sound alike


Tall man letter

11 Machine improve patient compliance with medication


E pill reminder

12 First thing before starting treatment for smoking patient


Determination of patient to stop smoking

13 Distribution in emergency
Automated dispensing

14 Case in hospital taken wrong drug and wrong amount and wrong administration method
what type of error
despising error

15 How can you make sure that the patients understand his medications
By asking the patient to repeat and explain the information given to him

16 Pharmacist prescribe benzoyl peroxide and soap to patient for acne after few days cause
worsen what pharmacist do?
continue benzoyl and soap

17 The most error that may occur when a patient is transferred from one unit of facility to another ?
Missing drug he was taking
18 Unit dose system in hospital ?
to decrease medication error

19 In 1997 , mean age of population was 50 year . In 2020 the mean age of population become 47 year
, What does that mean ?
Increase mortality

20 Nurse gave dose morphine to patient in the morning then forgot next dose how to prevent his
error ?
Medical reconciliation

21 to prevent medication error ?


Unit dose system
Cholesrol

1 Risk of myopathy due to which drug


Statin drugs

2 When to take statin?


statins work better when taken with a meal
Option (after dinner)

3 Gemfibrozil + statin
Sever myopathy

4 Which statin taken every night?


Simvastatin

5 Which statin has the higher intensity?


Arorvastatin 40

6 Drug interaction with statin??

7 High intensity statin


Atorvastatin 40

8 Man has Hyperlipidemia which consider high dose


Atorvastatin 40 mg

9 High cholesterol and triglyceride what is the best medication


simvastatin > is the best for elevated level of total cholesterol considered First line agent

10 Man has hyperlipidemia which consider high dose


atorvastatin 40 mg

11 patient will take esomeprazole what advise u can give to him


take for one month

12 Drug act on triglyceride and cholesterol


Simvastatin

13 what to lab test should preform if patient had myopathy from statin?
Plasma creatine kinase

14 which of drug high intensity of statin

15 Drug make rhabdomyolysis When taken with statin


Gemfibrozil

16 Cholesterol is consisting of four fused hydrocarbon rings + with a hydrophilic hydroxyl


group at one end

17 Decrease myopathy in statin


Co enzyme Q 10

18 Lab monitoring of myopathy while use statin


reatinine kinase
19 drug decreases synthesis of cholesterol in liver ?
Simvastatin

20 Drug contraindicated with ischemic heart disease patient with dyslipidemia ?


celecoxib deficiency

21 cholestyramin use in ?
ttt hypercholesterolemia

22 Used in hyperlipidemic ?
Simvastatin

23 drug interact with statin:


Ketoconazole

24 pharmacological uses of cholestyramine is


anti hyperlipidemic

25 pharmacological benefit of cholestyramine


decrease LDL

26 Drug for hyperlipidemia not decreaseLDL


fibrate (Fenofibrates (gemfebrozil) acts on Triglycerides

27 LDL is responsible for transportation of:


Cholesterol

28 What is the different type of Statin intensity ?

29 Montoring rosvastatin ?
(CK ) Creatine kinase Baseline and if there is symptom myalgia
CVS

1 Which drug use for Case patient has ductus arteriosus


Iv indomethacin

2 cardioglycoside
Digoxin inhibits sodium-potassium atpase

3 Calculate CHADS2 score in patient with DM, HTN, 60year , history of HF?
DM=1, HTN=1, CHF=1
The total is = 3

4 Lithium drug-drug interaction


Reduced serum levels with carbonic anhydrase inhibitors, chlorpromazine, sodium containing
preparations, theophylline, urea. Enhanced hypothyroid effects with iodine.

5 One drug approved to reduce mortality in heart failure??


beta blocker

6 Responsible for generate impulse


SA node

7 Propranolol similar to which one ?


Nadolol

8 Food interaction with amidarone ?


Grapefruit

9 Which following have vasoconstriction effect?


Alpha 1

10 Which drug cause constipation?


Verapamil

11 Test of IHD?
troponins (I or T)

12 Vasodilator side effect?


Tachycardia

13 Any of these have more alpha activity


Epinephrine

14 Drug Disease interaction


Celecoxib and cardiac toxicity

15 Cardiac enzyme used in diagnosis of heart attack


tropinon enzyme

16 Patient with ISHD coronary heart disease has injury and severe pain
morphine (the best for treat moderate to severe pain)

17 Patient suffer from heart disease


Sildenafil or teradonifil for 4 hr after Lisinopril
18 The best OTC for heart failure disease with knee pain.
paracetamol > is The BEST with the presence of heart disease

19 What is the meaning of P wave in ECG


P wave = arterial depolarization
QRS = ventricular depolarization
T wave = ventricular repolarization
U wave = Note that arterial repolarization is within QRS complex

20 Which of drug should not be used in atrial fibrillation


Verapamil

21 Activation of alpha 2 adrenoceptor lead to


Flushing – decrease

22 Activation of alpha 1 adrenoceptor lead to


Flushing – BP

23 Drug accelerate HF
Gabapentin

24 Management of Acute gout attack in HF patient


Colchicines

25 doxorubicin side effect


Cardiotoxicity

26 Beta blocker for HF patient


Bisoprolol

27 Beta blocker intrinsic


Acebutolol (β1-selective antagonist) and pindolol (nonselective β-blocker)

28 C/I in transit ischemic attack


Prasugrel

29 non selective a blocker for extravasation


Phentolamine

30 Decrease mortality in HF
beta blocker

31 Side effects CCB


First-degree atrioventricular block and constipation are common dose dependent side
effects of verapamil

32 Absolute contra-indication for use fibrinolytic


stroke within 2 month

33 first line vasopressor used in septic shock


Norepinephrine
34 which drug contraindicated in CHF
Pioglitazone

35 reflex tachycardia of hydralazine treated with ?


Propranalol

36 medication cause long Qr and bradycardia?


propranolo

37 which of the following decrease heart rate ?


metoprolol (beta1 blocker)

38 Dopamine is used in cardiac shock as ?


Increase .force of contraction

39 which of the following conditions have non preventable adverse effect ?


tachycardia with some forgot his dose of anti hypertensive drug and took 3pills

40 Nitroprusside in congestive heart failure patient is administered by ?


slow i.v. infusion

41 the action of digoxin can reduced with ?


antacid and hyperthyroidism

42 about digitoxin , which is false ?


Oral bioavaiability 100%

43 phenytoin uses other than anti convelcent agent are ?


ttt irreqular heart beat&ttt painful nerve coduction (neuralgia)

44 Hydralazine, ?
vessel relaxation. It dilates arterioles more than veins

45 about digioxin what is not true ?


taken only orally

46 Which of the following beta-adrenergic agents is not short acting?


Salmetrol

47 To supply the myocardium with 02 shortly after myocardial infarction we give ?


Aspirin

48 all the following diazoxide side effects except ?


Dieresis

49 Digoxin action except ?


increase conduction velocity in AV node

50 Dobutamine is used as ?
In cardiac surgery
51 Food increase effect of ?
Prpranolol

52 Dopamine is used in cardiac shock as ?


Increase force of contraction

53 Which one has narrow therapeutic index: nifedipine therapeutic index?


Digoxin

54 Drug prevent Microalbuminuria ?


ACEI

56 Overdose of the following lead to prolongation of Qr and bradycardia ?


Propranol

57 which one has narrow therapeutic index: ?


Digoxin

58 streptokinase is used in ?
myocardial infarction

59 Verapamil inhibit metabolism of ?


Digoxin

60 phase 2 in cardiac mean that ?

61 phase 3 in cardiac mean that ?


repolarization due to k outflux

62 Dopamine is used in cardiac shock as ..


selective dilate renal and mesenteric (vasodilation)

63 Dopamine is used in cardiac shock as:


Increase .force of contraction

64 Relief pain due to MI (Myocardial Infarction).


Morphine

65 Which of the following cardiac glycoside doesn't occur naturally


Amrinone
66 Streptokinase used in ...
Myocardial infarction

67 Hydralazine acts on heart by


Relax smooth muscle

68 ACEI role in CHF


prevent ventricular remodeling

69 ACEi is contraindicated in
bilateral renal artery stenosis
70 Altepase used is
Acute coronary syndrome

71 Patient with heart failure take ARB+CCB+ thiazides diuretics and still uncontrolled Stage 4 heart
failure , What is the next intervention ?
Spironolactone.

72 What is the Medication that used as rhyme and rate control for patients with heart failure?
Preference

73 Which medication considered as Rate control ?

74 Which medication considered as rhythm control ?

75 75 year Patient have MI and gout he take chochicin to improve acute attack but not effective.
What is the next intervention ?
use ibuprofen. indomethacin. acetoameniphen.
acute gout amack include ibuprofen 800 mg three to four times daily or indomethacin 25 to 50 mg
four times daily.
Asthma

1 Side effect of albuterol


Common:
Tachycardia, Nausea , Pharyngitis , Viral gastroenteritis , Vomiting, Backache
Feeling nervous, Headache, Sinus headache, Tremor, Urinary tract infectious disease
Bronchitis, Cough, Nasopharyngitis, Pain in throat, Rhinitis , Sinusitis, Upper respiratory
infection, Pain.

2 The most side effect associated with ICS inhaled corticosteroids?


Oral thrush

3 To prevent asthma induced by exercise give?


SABA

4 Side effect of albuterol


Tachycardia

5 Side effect of inhalad corticosteroid


Oral thrush

6 Morphine causes respiratory depression by


Depression of respiratory center in brain

7 Case of an asthmatic patient e lab data results (all of them were in normal range except for
k level was lower than normal ) the abnormality in test results is due to
inhalation LABA

8 side effect of inhaler corticosteroids


ICS deposition on the oral and laryngeal mucosa can cause adverse effects, such as
oropharyngeal candidiasis

9 Patient has asthma and glaucoma treatment of glaucoma

10 patient has asthma from exercise what must be taken before exercise
SABA

11 acquired pneumonia treatment

12 Gold classification of COPD in case scenario ( choice was A-B-C-D)

13 Which can cause Cardiac death


LABA

14 Albuterol side effect

15 Volume of distribution of theophylline


0.3-0.7 L/kg. Plasma protein binding: Approx 40-60%.
16 Glaucoma treatment in asthmatic patient

17 Class of asthma that comes 6 times over the week with 3-4 awaking night time
Severe

18 Non pharmacological treatment of asthma


Breathing exercise

19 assess severity of asthma


daytime symptoms, nighttime awakenings, frequency of rescue inhaler use, and activity
limitations along with lung function and exacerbation frequency

20 drug used in moderate to sever asthma ?


Omalizumab

21 Treatment of patient with chronic wheezing and cough ?


Corticosteroid

22 Which drug is contraindicated with asthma ?


Beta 2 antagonist

23 Side effect of albuterol ?


Tachycardia.

24 Side effects of albuterol include which of the following?


Tachycardia

25 volatile substance given by inhalation ?


Halothane
26 mechanism of using salbutamol inhaler ?
exhale , shake the inhaler , put it between your lips then breath slowly

27 which of the following drug taken by inhalation?


Amyl nitrite or halothane

28 drug should be used with precaution in asthma ?


b2 antagonist

29 corticosteroids with lowest potency ?


Hydrocortisone

30 beta2 blocker and aspirin are contraindicated with ?


Asthma

31 Respiratory acidosis is caused by


CO2 retention

32 month-old baby with a history of premature birth and chronic lung disease is admitted to the
pediatric intensive care unit with respiratory distress requiring intubation; fever; and a 3-day history
of cold-like symptoms. A nasal swab is positive for RSV. Which one of the following is the best
intervention?

Intravenous fluids and supportive care

33 treatment for severe allergic asthma?


omalizumab used in Treatment of moderate to severe persistent asthma

34 The case of a child suffering from intermittent asthma What is the best treatment?
(salbutamol or albuterol) or Class (short acting B2 Agonist )

35 What is the device that used with asthmatic child?


Nebulizer
Immunization

1 Type of allergy from bennet butter


Type IgE

2 Ig for allergy
Type IgE

3 Which immunoglobulin found in serum predominantly?


Immunoglobulin G (igg)

4 First Ig released in the body


IgM

5 Allergy types

6 Immunosupressant cause hairsutism?


Cyclosporine

7 By which one antibodies are produced?


B lymphocytes

8 Passive immunity?
Immunoglobulin

9 Patient has eaten peanut and he develop angioedema and swelling so which type of
allergy?
Immediate allergy

10 Which one is true about cromolyn for prevention of allergic rhinitis?


Take it before contact with allergen , 1 to 2 weeks prior to allergen

11 Allergic hypersensitive?
IgE
12 Which one of these in blood
IgG

13 Which one of these have gamma globulin


IgG

14 Which following decrease immunity?


Cyclophosphamide

15 Type of immune globulin initiate when infection attack


IgM

16 Which type of immunoglobulin that caused an Immediate immunity ?


immunoglobulin E (IgE)

17 IG can’t reach to placenta


IGM
Regulatory

1 Credit of health institution is issued from which?


CBAHI central board for accreditation of health institution

2 Want to open pharmacy but no enough Saudi pharmacists to hire which is true?
MOH may make an exemption of nationality

3 Pharmacist saw misleading advertisement online what to do?


make report to MOH

4 If stealing happen to who the pharmacist must report


Pharmacy manager

5 Who is responsible from formulary


Pharmacy therapeutic committee

6 An employee perfected his job and the manger wants to something what will the
employee receive from his manger
Reward

7 You discovered steeling narcotic drug who you will call


Head of Pharmacy

8 If you are pharmacist and see in social media Advertising about medicine is misleading
what will do?
Report to SFDA

9 becomes controlled drug recently


Prigablin

10 Original copy of the controlled medication given to :


Pharmacy

11 Room temperature
68 to 77°F (20 to 25°C)

12 Freezer temperature
-10°C to -25°C

13 Refrigerator temperature
2°C to 8°C

14 storage of narcotic prescriptions according to Saudi national regulations ?


36 months

15 The task of pharmacists and medicine centers & hospitals ?


Control formulary of drugs

16 You should keep the prescription of narcotics in the pharmacy for ?


five year
17 The responsibility of pharmacist in hospital is ?
deal with drug interaction

18 Pharmacy and therapeutic committee ?


maintain of formulary of drug

19 Pharmacy and therapeutic committee


management of the drug

20 expiration of drug
management of the drug

21 Temperature?
room 15 -25 , Warm and Cool 8-15

22 What is the narcotics included in the green list?


Fenethylline , Methaqualone

23 Chronic narcotics medications can prescribe for ?


30 days

24 Who is responsible to stock the unit?


Nurse

25 Who is responsible for stock medication in nurse ward?


Nurse

26 Who is responsible for stocking in nurse ward ?


Pharmacist

27 The original prescription of narcotic must kept in ?


Pharmacy

28 Diazepam Prescription valid until ?


7 days

29 The maximum allowed amount of diazepam to dispense?


30 days

30 oxycodone prescription Valid until ?


7 days

31 If we have a red Color prescription, That’s mean this prescription is Valid for?
7 days

32 What is the function of Pt commimee


drug formulary

33 Concor and (another drug) are look alike what we recommend to use in a purpose of decreasing
error :
Sticker blue
34 Vincristine fatal when given intrathecal what the process make safe from this error while
dispense to nurse ?
dispense drug in piggyback

35 Hazard medications
Hazardous drugs are:
Teratogenic
Carcinogenic
Genotoxic (damage the
DNA and can cause
Cancer)
Have reproductive
Toxicity
Cause organ toxicity at
Low doses
Hazardous drugs (hds) can be roughly divided into three categories:
Antineoplastics (chemotherapy drugs, which are the majority of hds), non-antineoplastics
(many, such as hormones and transplant drugs), and drugs with reproductive risk. Spilling
An antineoplastic on bare skin can cause severe skin irritation; methotrexate tablets will
not.
Septic shock

1 In patient with septic shock what is the most important step?


Administer fluid

2 Septic shock 1st line?


Norepinephrine IV infusion

3 septic shock patient with normal sodium value with high potassium and creatine the treatment is ?
dopamine
Cardiac shock = Dopamine Anaphylactic shock= Epinephrine
Septic shock= 1.intravenous fluid (normal saline) first 2.Dopamine

Septic shock in this question with normal sodium level and renal failure (kidney injury)= Dopamine

4 which of the following is contraindicated with angina ?


Vassopresin

5 A patient purchasing sublingual nitroglycerin tablets should be told to store the medication ?
in an amber glass bottle with a metal cap

6 Tolerance is a problem when using Nitroglycerin, which of the following is true ?


Dosing schedule may affect tolerance

7 Role of B-blocker for angina ?


Prevent reflex tachycardia

8 Sign of shock ?
Hypoperfusion

9 Patient has septic shock and his BP 70/40 mmHg , a slight increase in K level and Na within normal
range .. which drug should be treated with ..?
Saline

10 What is the Patient education regarding nitroglycerin?

- Under tongue Store in light resistance container with a metal cap


- Keep away from the heat
- Administer one tablet under the tongue or in the buccal pouch at the first sign of an acute anginal
attack. Allow tablet to dissolve without swallowing.
- One additional tablet may be administered every 5 minutes until relief is obtained.
- No more than three tablets are recommended within a 15-minute period.
- If the pain persists aier a total of 3 tablets in a 15- minute period, or if the pain is different than is
typically experienced, seek prompt medical amention

11 Case patient with septic shock , All lab result normal except creatinin 300 high what is the best
treatment ?
Give dopamine
Herbal

1 Patient suffer from stress and want to improve endurance take?


Chamomile

2 Which glycosides of senna responsible for cathartic effect?


Anthraquinone

3 Which one is true about herbal supplement?


Need good manufacturing practice GMP

4 Extraction of active material from plant by adding solvent and heating?


Decoction

5 What herbs for mental disease?


Gingko biloba

6 plant source use for acute gouty


Colchicine

7 Cranberry used for


UTI infection

8 oil used in sunburn


Aloe

9 cyanide toxicity ?
cardiac toxicity

10 Senna is considered which type of laxative ?


It stimulates muscle movement in the intestines

11 Herbal for sedative ?


Valrein

12 case patient with sun burn ?


oleo vera

13 Glycoside in senna ?
Anthraquinone

14 Used in dark box


Clove Oil
Economics

1 Cost types

2 In cost utility analysis which one is the outcomes?


Adjusted quality of life

3 What is the benefit of pharmacoepidemiology


Utilization of drug

4 The economic study for life adjusted year quality?


CUA

Know the difference between direct medical, non-medical, indirect and intangible costs

cost of medicine and cost of loss productive


Direct and indirect
Pharmacology

1 neuromusclar blocker block ?


Acetylcholine

2 flutamide is used for treatment of ?


prostate cancer

3 propylthiouracil cause ?
Agranulocytosis

4 in ovulation phase hormone which predominant ?


LH

5 which of the following drug increase the incidence of BPH ?


Chlorpheniramine

6 recommended dose of calcium in geriatric ?


1200 mg

7 recommended vitD3 dose for geriatric?


600IU

8 human chorionic gonadotropin is used to ?


Induce ovulation and treatment of infertility

9 gray man syndrome caused by ?


Amiodarone

10 treatment of myasthenia gravis by ?


pyridostigmine or neostigmine

11 Which metal used in treatment of rheumatoid arthritis ?


Gold

12 The antimalarial to be avoided in glucose-6-phosphate dehydrogenase deficiency ?


Primaquine

13 Coal tar uses in ?


(psoriasis)

14 isoniazid used in ?
Tuberculosis

15 sevelamer used in patients with chronic kidney disease for ?


Hyperphosphatemia

16 which of the following indicator for a toxicity of a drug ?


necrosis of liver by acetaminophen

17 Nitro glycrin side effect is ?


Headache
18 which of the following is less potent than tubocurarine ?
Succinylcholine

19 ergot alkaloid used in ?


Uterine stimulant

20 phenylephrine used in ?
Decongestion

21 which of the following can be used in treatment of hyperthyrodism ?


potassium iodide chloride

22 Women came to the ER with pointed pupil. With vomiting and dizziness... Which drug is
responsible?
Heroin (Heroin because it is opoid agonist and cause miosis)

23 Man came to ER vomiting and dizzy.after he ingested toxic dose of certain drug Which is the most
important step?
Watch the vital signs and Make them normal

24 acetylcystiene is ?
Mucolytic

25 drug that makes urine red ?


Rifampicin

26 Hydroxyurea is antineoplastic drug is ?

27 drug make urine red other than rifampicin?


phenazopyridine (Pyridium), and laxatives containing senna

28 syncop may occure with the 1st dose of the folowing ?


Prazosin

29 the main cause of digitalis toxicity is ?


renal faliure hypokalimai is main cause of digitalis toxicity

30 process require CYP450 ?


Oxidation

31 Adsorption, which is not true ?


Chemical property and Irreversible

32 one of following is not alpha 2 agonist ?


Guanethidine

33 about side effect of quinidine ?


Cinchonism

34 guanthedin SE are all ?


orthostatic hypotension
35 mizolastine used as ?
antihistaminic non sedating

36 Various opiates may be used as all of the following except ?


Anti-inflammatories

37 Which of the following action is not seen with sympathomimetics ?


Pupil constriction

38 Cyclosporine is used for ?


prevention of transplant rejection

39 A hospital on formulating drugs, Efficacy, Work overload, and Costs are taken in concern. A, B, C, D,
and E are antifungal drugs available in the market. AlI have the same efficacy. The hospital was
using drug E for a period of time. According to the following data which drug is best to be used by
the hospital: DRUG COST FREQUENCY DURATION ?
2.25 BID 7 DAYS

40 Which not effect of caffeine ?


skeletal muscle relaxant

41 which of the foll. drugs show most multidrug resistance ?


Antibiotics & anticancer

42 Concerning COX-2 inhibitor, which is not true ?


combination with non-selective agents provide additional benefit

43 Which of the following statements describing first dose phenomenon that associatedwith prazosin
is true?
A marked postural hypotension 30 to 90 minutes following the initial dose of prazocin.

45 Milironin has following except ?


increase ca intercellular

46 Phenylepherine ?
used in nasal decongestant & in high doses stimulate b adrenergic receptor

47 One of the statement considering Tetracaine One of neuromuscular blocker?


The most one of choice in spinal anaesthesia

48 Neuromuscular blocker is used of choice in renal and hepatic failure ?


Atracurim

49 All of these are antichlonergic except ?


Mebeverine

50 Neuromuscular blocker has the lowest t half ?


Succinylcholine

51 May used as anti-spasmodic ?


Baclofen
52 Rapid relief of acidity ?
Al hydroxide

53 A drug for hyperthyroidism associated with a agranulocytosis is ?


propyl thiouracil

54 Bioavailability refers to the extent and rate at which the active moiety (drug or metabolite) enters ?
systemic circulation ( the site of action)

55 liquid dosage form differ from solid in ?


ease to administration

56 Drugs act on non-receptor mechanism?


Deferoxamine

57 Anti-inflammatory MOC ?
decrease prostaglandin

58 The concomitant administration of H2 blockers may ?


increase the dissolution rate of enteric-coated naproxen

59 sevelamer is to treat ?
Hyperphosphatemia(Renagel)

60 Side effect of diphenhydramine ?


Drowsiness

61 Doses qusetions Vitamin D & Calcium & Folic acid ?

62 Drug that is susceptible to cause drug dependence ?


Benzodiazepines

63 A drug is contraindicated with patients have G6PD deficiency suffering from malaria ?
Primaquine

64 Magnesium is contraindicated in renal impairment because of ?


Accumulation

65 Digitalis toxicity due to ?


Hypokalemia

66 LDL indicate ?
Arteriosclerosis

67 benzyl benzoic acid use ?


for ACNE

68 Use of combination OCs ( Oral contraceptives ) contraindicated in case of


DVT ( Deep vein thrombosis )
69 A woman had DVT and was treated a year ago , now she wants to use oral contraceptive pills the
most suitable ocs for her ?
Levonorgestrel ( or l - norgestrel or D - norgestrel )

70 which of the following may cause acute renal failure ?


IV radiological substance

71 Early symptoms of aspirin poisoning are ?


ringing in the ears & blurred vision

72 Obese patient what the effect of obesity on absorption of lipid soluble drug ?
no effect

73 woman taking oxycarbamazipine , after 2 weeks of administration , redness and flushing appear ?
keep using oxycarbamazipine

74 Least glucocorticoid potency


Hydrocortisone

75 Signs of hypothyroidism except?


Tachycardia

76 Which cause hyperthyroidism?


Amiodarone

77 Drug treat acute mania?


Lithium

78 Which of the following more frequent


Mode

79 Best test for thyroid


TSH

80 What is hypothyroidism
T4 low and TSH high

81 Management of hypothyroidism
Levothyroxine

82 Drug choice for BPH


Tamulsocin

83 What is best advice for travel diarrhea


Hygiene of hand

84 Used in prophylaxis of Migraine


Beta blocker Propranolol is effective in reducing migraine episodes
When used prophylactically

85 Man has BPH treated with


tamsulosin and finasteride
86 when transmission of body to anther to improve immunosuppressant
Cyclosporine

87 drug use for allergic eye

88 OTC in treatment of ear

89 First line for migraine


Triptans

90 Treatment for open glaucoma


Prostaglandin analogue

91 Patient develop involuntary movement face which drug patient take


Haloperidol

92 Immediate relive allergy of rhinitis after house cleaning


Chlorpheramine

93 Before start doxorubicin check

94 Which one is selective 5 lipoxygenese inhibitor?


Zeliuiton

95 Drug cause retinopathy


Hydrocychlorquin

96 Drug that used in Open angel glaucoma


Latanoprost

97 main side effect of nitrates ?


Headache

98 Side effect of atropine


urine retention

99 BPH is worsen by ?
anticholinergic drugs

100 selective beta blocker for treatment glucoma:


Betaxolo

101 A 12 years old girl suffer from rashes after treating with oxycarbamazepine
shift to ethosuxamide

102 A drug induces Water retention


indomethacin

103 A drug with narrow therapeutic index


Phenytoin
104 Which of the following drugs are considered drugs dependence in normal dose ???
fluxetine (Prozac)

105 A drug has Dose-dependent according to pharmacokinetic parameter


Phenytoin

106 Naloxone exert its action through ...


Antagonism

107 A patient who is taking sevelamer drug means that he / she suffers from
hyper phosphatemia

108 which of this groups indicate pharmacological toxicity


liver cirrhosis and paracetamol

109 drug induced BpH


Chlorpheniramine

110 Dopamine causes:


elevate blood pressure

111 Cimetidine is
enzyme inhibitor

112 side effect of atropine is


urine retention

113 drug and disease interaction:


Furosemide and osteoporosis

114 Increase of BPH is ...


Chlorpheniramine

115 A 12 years old girl suffer from rashes after treating with
oxycarbamazepine
shift to ethoxsumide

116 rapid relief of acidity ...


Al hydroxide

117 A drug Treats Bph by relaxation of bladder neck?


Terazocin

118 Decreases effect of phenytoin?


Alcohol

119 a pregnant woman in her third week she take levothyroxine 100 mcg
, you advise her:
increase the dose of levothyroxine

120 The lowest in potency


Hydrocortisone
121 which of the following drugs cause dependence
Benzodiazepenes

122 gray man syndrome due to


amiodarone
Gray baby syndrome ----> Chloramphenicol induced
Gray man syndrome ----> Amiodarone induced
Red man syndrome ----> Vancomycin induced
Lupus like syndrome ----> Hydralazine & Procainamide induced

123 Cardiotoxicity is caused by:


Celecoxib

124 Cyclizine is used for:


nausea and vomiting

125 uses of promethazine:


Nausea & Vomiting

126 SSRI fluoxetine used as ---


mood stabilizer

127 Hypothyrodism is caused by:


cushing syndrome

128 Ophthalmic preparation should have these properties except:


should contain preservative sitotosc

129 Quinidine is similar in its action to?


Procainamide

130 What is the the Side effect of ethambutol?


optic neuropathy

131 What is the Anti cancer drug that cause cardiotoxicity?


Doxorubicin

132 What is the Patient education for levothyroxine?


take it on an empty stomach, 30 to 60 minutes before breakfast. with a full glass of water

133 What is the preferred Treatment of glaucoma in asthmatic patient?


Betaxolol, latanoprost

134 Which Medication used anti-malarial in Saudi Arabia?


Primaquine

135 Case pt take selgiline stoped 2 week and complain of dry cough ?
Dextro reasonable choise

136 Which Hormones are produced by posterior pituitary gland?


Vasopressin and oxytocin
137 Which drug are used in treatment of multiple sclerosis?
Natalizumab

138 Which drug that used in treatment of hypothyroidism ?


Levothyroxine

139 Which drug that used in treatment of hyperthyroidism ?


Methimazole

140 What is the patient counseling with levothyroxine ?


Patients are advised to take their levothyroxine in the morning, at least 30 minutes to hour before
eating, on an empty stomach and with only water

141 If patient takes diphenhydramine which the most advice ?


This drug cause Drowsiness, dizziness

142 ROLE OF BB IN Hyperthyroidism?


Propranolol is the preferred agent for β-blockade in hyperthyroidism and thyroid storm due to its
additional effect of blocking the peripheral conversion of inactive T4 to active form T3.

143 Patient take aldosterone what is factor effect in clearance?


impairment Renal

144 Max amount of individual dose we can take it intramuscular in gluteus muscle?
1ml

145 What is the treatment of Athletes foot ?


Terbinafine topical

146 Medication for refractory multiple sclerosis?


Teriflunomide or natalizumab

147 Pseudoephedrine is much preferred than ephedrine ?


Pseudoephedrine have less CNS side effect than ephedrine

148 What is the treatment urinary emergency?


antimuscarinics such as oxybutynin and tolterodine, aim to relax the bladder. Different medications
will help with different causes. For example, if the cause is a urinary tract infection, a course of
antibiotics.

149 What we have to check before initiate canagliflozine?


Renal function

150 Zolpidem female dose ?


Maximum dose: 5 mg for females, 10 mg for males

151 Antihistamine and 5-hydroxytryptamin antagonist?


Cyproheptadine

152 Anti plamelate fibrinoletic ?


Streptokinase
153 Amiodaron Side effect :
Pulmonary febrosis

154 Eclampsia management ?


Mg sulfate

155 Type of prostaglandin that inhibited by aspirin?


Aspirin is non-selective and irreversibly inhibits both forms (but is weakly more selective for COX-1).

156 Tegretol is the option in treatment of :


Neuralgia

157 safest drug to used in sulfa allergy :


Frusemide

158 dose of zolpidem in female


5 mg

159 aminophilline and theophylline ratio


85%

160 facial edema with


Captopril

161 levodopa / carbidopa depend on what ?


Carbidopa

162 levodopa / carbidopa counselig ?


On empty stomach

163 lithium monitoring ?


Renal

164 Pilocarbine
Ophthalmic , Open-angle glaucoma
Oral
Dry mouth after radiotherapy
Oral
Sjogren's syndrome

165 Treatment of Mania


Lithium

166 patient have severe acne what can she use


Isotretonin
HIV

1 nevirapine is used for ?


treatment of HIV (AIDS)

2 Don't give vaccines to ?


Patients with CD4 < 200

3 To know the severity of immunodeffiecency in HIV ?


CD4 count

4 Analogue for HIV ?


Zidovudine

5 Antiviral for HIV which nucleoside reverse transcriptase inhibitor ?


Zidovudine

6 HIV patient with no symptoms and CD4 count is 555 when he should receive vaccine ?
He must avoid MMR and varicella vaccines if CD4 count less than 15% (less than 200 cells/mm3 if
older than 5 years)

7 HIV medication for pregnant ?


Zidovudine
HTN

1 Treatment of hypertension in black patient?


thiazide-like diuretic or long-acting dihydropyridine calcium channel blocker

2 How to start anti- hypertension in elderly?


Start low dose and increase gradually

3 Patient with BPH has urine incontinence and take oxibutinin so what is the best approach
Afluzosin

4 Patient has hyperkalemia treatment ?


Furosemide

5 Patient with African descent has hypertension so first line?


Amlodipine

6 Patient has allergic rhinitis doctor prescribe pseudoephedrine you found that he has
history of hypertension what to do?
Don’t dispense the prescription – use pseudoephedrine with caution if controlled
Hypertension

7 Dry cough cause by?


ACEI

8 Dash Diet lifestyle of disease ?


Hypertension

9 Drug use in Raynaud's syndrome ?

10 Aliskarin which class?


Direct renin inhibitor

11 What is component responsible of cough in ACE inhibitor


Bradykinin

12 Spironolactone caused increase in potassium how to decrease


Use thiazide because it decrease potassium

13 What is the best diuretic in heart failure with renal insufficiency


Loop diuretics (Bumetanide, furosemide, torsemide, and ethacrynic acid)

14 Drug cannot be used as monotherapy in mild hypertension?


Hydralazine

15 first line treatment of hypertension in adult ?


Amlodipine

16 Which drugs not true companions


Valsartan and Lisinopril

17 patient use this cycloserine and amikacin drugs for 6 months and he loss hearing Which
drug case that
Amikacin
18 Hypertension agents that cause depression

19 Furosemide infusion over how much time required


40 mg via slow inj over 1-2 minutes

20 Pt with Hypertension + DM

21 The mechanism of action of enalapril?


block the enzyme ACE which cleaves angiotensin I to form the potent vasoconstrictor
angiotensin II

22 α1 receptor activation lead to


elevate systolic B.P

23 antihypertensive drug avoided in elderly


Amlodipine

24 the main side effect of alpha antagonist (blocker) as prazosin is ?


Postural hypotension (orthostatichypotension)

25 loop diuretics (furosemide) cause ?


ototoxicity ( deafness)

26 beta blocker with intrinsic sympathomimetic activity ?


pindolol and acebutalol

27 deltiazem used in hypertension to ?


inhibit calcium influx

28 diuretic of choice in renal failure ?


loop diuretic

29 amikacin cause which of the following adverse effect ?


nephrotoxicity(kidney)

30

The answer of table :


Drug A is anti renin (rennin blocker)
Drug B is ace inhibitor
Drug C is ARB(angiotensin receptor blocker) Drug D is diuretic NOTN
31 The most dangerous adverse effects of Amikacin is ?
(kidney disease)

32 antihypertensive produce tackycardia ?

33 Captopril and Enalapril do all the following except ?


Competitively blocks Angiotensin Il at it's receptors

34 The aim in the management of uncomplicated hypertension ?


< 130\80

35 indapemide used in ?
Pulmonary eodema Essential hypertension

36 which one causes hypotension due to blockage of efferent limb ?


bretylium

37 Which one of the following cause hypertensive crisis if withdrawn suddenly ?


Clonodine

38 Indapamide used in ?
Essential hypertension

39 long long term treatment with thiazide requires ?


K

40 A Drug causes sodium and water retention ?


Indomethacin

41 Which of the following induce diarrhea ?


Indomethacin

42 B_blockers are contraindicated for hypertensive patient in case of ?


Asthma

43 Drugs induced cough Except ?


Losartan

44 ACEI contraindication ?
renal failure

45 Drug used in treatment for Hypertension and migraine ?


clonidin or probranolol

46 Class of drugs can transform macroalbuminurea to microalbuminurea:


ACEI

47 African American women take lisinopril and another hypertensive drug suffer from nose swelling
and other symptoms what make her more suspected to this reaction
Ethnicity
if no ethnicity in choices .. choose Drug combination
48 which of the following true?
ACEi cause dry cough

49 A Drug decrease Na & k and increases uric acid reabsorption


Hydrochlorothiazide

50 antihypertensive drug act by increase of urination


Hydrochlorothiazide

51 drug alpha 2 agonist cause


decrease hypertenstion

52 side effect of thiazide diuretic:


Hypokalemia

53 drug increase excretion of NA, k & increase reabsorption of uric acid


Hydrochlorothiazide

54 Furosemide 120mg infuse during?


A maximum infusion rate of 4 mg/minute has been recommended for doses greater than 120 mg

55 Case: age of pa3ent 60 years have HTN,DM and CHF , calculated CHADS2 ?
CHADS2 Score 3

56 example of k sparing diuretic?


Spironolactone

57 Case patient with HTN ,takes lisinopril and carvedilol , His lab result : HR 55
BP 70/100 what is your recommendation ?
reduce dose carvedilol
Study design

1 What Is Strongest Evidence Based Study?


Meta-analysis

2 Which book is about Compounding


USP

3 Strongest evidence study


Evidence from a systematic review or meta-analysis

4 Case series?
Case series depend on more than one patient with a similar experience or many case
reports combined into a descriptive review

5 Case report?
Case report depent on one patient

6 Rare disease , new disease which study is best?


Case series and case reports

7 Case study focus on specific time which one?


Cross sectional

8 Which one of case studies include intervention?


RCT

9 Person want to study the effectiveness of vancomycin in reducing C.diff which types of
studies he will be using?
Case report if single patient
Case series if a few patient

10 what is the type of study design were the author collects the results of high quality articles
and analyzed it to have one summary statistic?
meta-analysis

11 disadvantage of RCTs
Expensive

12 Study design is using for prevalence?


cross sectional

13 study in which we use different studies to make a conclusion ?


meta analysis

14 Studies in which data collected from different studies


systematic review

15 What is the type of Study design for smoker and non smoker at the same period of time ?
Cross sectional

16 What is the study design that used for rare disease?


Case control
17 What is the Study design that describe something happen in the past ?
Retrospective.

18 What is the study subject is control group arm active ?


Cross over

19 study the control is the same of active arm


cross over
MOA

1 Digoxin
inhibits sodium-potassium atepase
Na+/K+ ATPase inhibitor

2 Donepezil
reversible acetylcholinesterase (ache) inhibitor

3 Lestyramine
inhibits enterohepatic reuptake of intestinal bile salts and thereby increases
the fecal loss of bile salt-bound low density lipoprotein cholesterol

4 Minoxidil
Vasodilator act by producing relaxation of vascular smooth muscle, primarily in arteries
and arterioles

5 Amantadine
Anti-Parkinson Agent inhibiting the N-methyl-D- aspartate (NMDA) type of glutamate receptors.

6 Chylomicron
Transfer lipids or fat from intestine to peripheral tissues.

7 Cyproheptadine
Antihistamine and serotonin antagonist

8 clonidine
Stimulate alpha and decrease sympathetic responses

9 Dantrolone
Dantrolene, a direct-acting skeletal muscle relaxant, inhibits the release of Ca ion from the
sarcoplasmic reticulum leading to decreased response to action potential and decreased
muscle contraction.

10 Atropine
is an anticholinergic agent(antimuscarinc agent ) which competitively blocks
the muscarinic receptors in peripheral tissues

11 TCAs
The TCAs block norepinephrine and serotonin reuptake into the presynaptic neuron
Blocking of receptors: TCAs also block serotonergic, α-adrenergic, histaminic, and
muscarinic receptors

12 Duloxetine
SSRI

13 Fingolimod
Is used for treatment of multiple sclerosis (MS) it’s an oral drug that alters lymphocyte
migration resulting in fewer lymphocytes in the CNS

14 Benzodiazepine
15 Aspirin
Aspirin is a weak organic acid that irreversibly acetylates (and, thus, inactivates)
Cyclooxygenase

16 abciximab
abciximab inhibits the GP IIb/IIIa receptor complex. By binding to GP IIb/IIIa, abciximab
blocks the binding of fibrinogen and von Willebrand factor and, consequently, aggregation
does not occur

17 L-dopa
Levodopa increases dopamine levels in the brain leading to the stimulation of dopamine
receptors.

18 Clomifene
Clomifene is a nonsteroidal compound that has both oestrogenic and anti-oestrogenic
effects. It stimulates ovulation by inhibiting the negative feedback effect of oestrogens at
receptor sites in the hypothalamus and pituitary, thereby increasing hypothalamic GnRH
secretion w/ subsequent release of pituitary FSH and LH

19 Co enzyme Q 10
Coenzyme Q10 (CoQ10) is an antioxidant that your body produces naturally. Your cells use
CoQ10 for growth and maintenance.

20 carbon monoxide
Carbon monoxide poisoning occurs when carbon monoxide builds up in your bloodstream.
When too much carbon monoxide is in the air, your body replaces the oxygen in your red
blood cells with carbon monoxide

21 clopidogrel
inhibit ADP that cause platlet aggregation

22 Cromolyn
mast cell stabilizer Released based on inflammatory substance

23 Salbutamol
salbutamol is a short-acting, selective beta2-adrenergic receptor agonist

24 Calcium channel blockers


Calcium channel antagonists block the inward movement of calcium by binding to L-type
calcium channels in the heart and in smooth muscle of the coronary and peripheral
arteriolar vasculature. This causes vascular smooth muscle to relax, dilating mainly
arterioles. Calcium channel blockers do not dilate veins.

25 Minoxidil
Vasodilator

26 zyrtec
h1_blocker (zyrtec (cetirizine)

27 bisacodyl
Stimulation of enteric nerves to cause colonic contraction
28 bulk laxatives
Add water and bulky to stool and soften stool like a jelly

29 cyclosporin ?
immunosuppressant after organ transplantation to reduce possibility of rejection of new organ by
immune system

30 both of cimetidine(h2blochker) and spironolactone ?


anti androgenic effect

31 choloroquine ?
choloroquine bind to heme and prevent it's polymerization to hemozin

32 benzodiazepine action ?
Inhances GABA inhibitory effect ( gamma - Aminobutyric acid GABA agonist )

33 Diltiazem actin ?
CCB cacuim channel blocker

34 MOA of . spironolactone ( Adverse effect of Spironolactone )


Hyperkalemia

35 MOA of beta - carotene?


Precursor for vit A ( retinol ) Antioxidant

36 MOA of antiarrhythmic Class 1A ( quinidine ) ?


decrease rate of phase o depolarization

37 MOA of cromolyn ( cromolyn sodium ) ?


mast cell stabilizers

38 Atropine is ?
Muscarinic antagonist

39 Mechanism of action of Dopamine ?


Dopamine b1 agonist

40 The drug is most commonly used in the selective COX2 inhibitors ?


Celecoxib

41 Anti-inflammatory MOC ?
decrease prostaglandin

42 Clonidine MOA in treatment of Hypertension:


Alpha2 Agoinst decrease sympathy outflow

43 Allopurinol ?

Anti-inflammatory

44 amphetamine pharmacological action ?


indirect acting adrenergic agonist.
45 Calcium-channel blockers mode of action ?
decrease the inward calcium to cells

46 about minoxidil ?
dilate arteries only

47 dantrolene ?
cause skeletal muscle relexant bybinding to the ryanodine receptor decreasing intracellular calcium
concentration

48 has 5-HT antagonist and H1 antihistamine effects ?


Cyproheptadine

49 phentolamine ?
alpha antagonist

50 florouracil ?
is Pyrimidine derivative

51 loperamide ?
opioid agonist anti-diarrheal

52 clomiphene ?
inhibit negative feedback of estrogen

53 amphetamine ?
indirect acting adrenergic agonist.

54 Cephalosporin act ?
Inhibitors of cell wall synthesis

55 digoxin ?
+ve inotropic effect

56 prazosin ?
is post synaptic alpha1 blocker

57 aminglycoside ?
protein synthesis inhibitor

58 rofecoxib is ?
selective cox2 inhibitor

59 clonidine and methyldopa are ?


alpha 2 agonists

60 loperamide stimulate ?
MU receptors

61 atenolol is ?
selective beta 1 blocker
62 morphine act on ?
mu receptors

63 clomiphene ?
Inhibit negative feed back of estrogen

64 aminoglycosides ?
protein synthesis inhibitor

65 bethancol ?
Selective muscarinic agoinst

66 Flutamide ?
Used in treatment of prostate cancer which act as Nonsteroidal
antiandrogen that inhibits androgen uptake and / or inhibits binding of
androgen in target tissues

67 domperidone & doxazocin


Domperidone has peripheral dopamine receptor blocking properties (dopamine antagonist) and
does not readily cross the blood-brain barrier.
Doxazosin Competitively inhibits postsynaptic alpha1-adrenergic
receptors

68 benzodiazepine ?
GABA agonist

69 statin
Decrease cholesterol by HMG coA reductase enzyme inhibitor

70 Fluoxetine
selective serotonin reuptake inhibitor (SSRI)

71 Benzodiazepine?
Anxiolytic

72 Diazepam action is
increase glutamate secretion

73 Labetalol
is a selective alpha-1 and non-selective beta adrenergic blocker

74 (Isoprenaline)
Non selective β stimulant

75 Amlodipine
Calcium channel blocker (CCBs)

76 Phenylephrine act on
alpha agonist

77 amantadine
antiviral and antiparkinsonian
78 Ranitidine mechanism of action:
H2 antagonist

79 Cyproheptadine acts as:


Antihistamine

80 Mechanism of action of TCAs is


decrease reuptake of amine at synaptic

81 nitroglycerin
coronary vasodilation

82 dantrolene
Postsynaptic muscle relaxant inhibitors ca²⁺ ions release

83 Fluroquinolone
Fluoroquinolones act by inhibiting two enzymes involved in bacterial DNA synthesis, both of which
are DNA topoisomerases that human cells lack and that are essential for bacterial DNA replication,
thereby enabling these agents to be both

84 cromolyn
Cromolyn is a mast cell stabilizer that inhibits the Type I immediate hypersensitivity reaction by
preventing the antigen-stimulated release of histamine. also prevents the release of leukotrienes
and inhibits eosinophil chemotaxis

85 Asprin antiplatlet mechanism ?


irreversibly inhibits prostaglandin H synthase (cyclooxygenase-1) in platelets and megakaryocytes,
and thereby blocks the formation of thromboxane A2 (TXA2; a potent vasoconstrictor and platelet
aggregant).

86 abixcimab
glycoprotein IIb/IIIa receptor antagonist
Drug information

1 Phase 1 of drug development


pharmacokinetics study in human

2 Review article is
secondary resource

3 Color drug references

4 USP 797
Sterile

5 Which the following have error in prescription


Abbreviation

6 What is the lowest level of evidence in the evidence pyramid


Idea and opinion

7 You are having a study that you want to represent the number data on it what
Is the best way
Quantitive

8 Type of error when you discovered mistake drug before reaching to patient
category B

9 Name of system that help doctors in make decisions for choice the drug
health theoretical model

10 pharmacokinetic in which phase


0
11 Small sample which phase
1
12 Large sample which phase
3
13 Pharmacovigilance which phase
4
14 Side phase
2
15 When the pharmacopeia put the drug as phase 4 ?
Safety

16 how many phases applied to drug to be approved by FDA ?


4 phases

17 if a medication error happened, the first person to refer to ?


Prescriber

18 Benefits of using the unit - dose system ?


To decrease medication errors ( dose errors

19 Type of pharmacist salary?


Direct medical

20 Committee consists of five members


IRB institutional review board

21 Example direct medical cost and an indirect cost

22 USP General chapter

23 Directly decrease medication errors ?


Mandatory reporting

24 Medication characterized by low cost fast moving ?


Bulk medication

25 If the doctor ask the patients about side effects , these considered which type of pharmacovigilance
?
Active pharmacovigilance

26 Mission and vision , which type of planning?


strategic planning

27 example of ethical principles


Justice

28 tertiary resources example ?


Review article

29 patient said : “ i have pain ..... “ this considerd part of :


Subjective
30 tertiary recourse that arranged drugs alphabetically, their compatibility , stability and
administration ?
Handbook for injectable drugs
Pharmacutics

1 What process hinder drug from dissolution


Make tablet more compressed

2 Issues of large tablets


delay dissolution

3 What is the basis of pharmaceutics for drug classification?


solubility and partition coefficient

4 Which one prevent tablet dissolution?


Hard compressing tablets

5 grinding of solid or powder in liquid or ointment is ?


LEVIGATION

6 When we decrease particles size?


In limited dissolution substance

7 Drug with high Distribution Volume, What Means ?


conc. of drug in tissues is higher than that in plasma

8 A fixed dose of drug that follows 1" order elimination depends on ?


dose of drug

9 45 year old man his volume of distribution is 35 L what the interpretation ?


drug is highly distributed in.plasma

10 Plasma conc. Of drug in all body is ?


rate of distribution

11 PKa for normal water in room temperature ?


14

12 Characters of liquid dosage form


Fast in absorption

13 Aspirin Pka will be stable at PH 3.2


2

14 A fixed dose of drug that follows 1" order elimination depends on ?


dose of drug

15 Which is not considered in formation of suspension


Use chelating agent

16 Flavor used in drugs


Fruit

17 All factors affect the distribution of drugs except


type enzyme response from metabolism
18 45 year old man his volume of distribution is 35 L what the interpretation ?
drug is highly distributed in.plasma
1 measure crcl in a case
Cockcroft- Gault

2 after drug metabolism the drug become ?


Polar

3 albumin and dextran and starch are ?


colloid solutions

4 Most source of infection in sterile area is


People

5 1Ib = ????
1Ib = 454 gram

6 scale for adverse drug reaction with drugs ?


Narnjo scale

7 Oleaginous base for ointment?


white petroleum,

8 To estimate Cr.cl in pediatrics we use?


Schwartz equation

9 Normal plasma in human body in liter


35 L

10 function of coenzyme Q ?
Reduce Nad to NadH

11 in buffer room type of gloves ?


sterile gloves , with powder

12 Narnjo scale for drug = 9 what does that mean ?


Definite

13 Equation of creatinine clearance in adult?


Cock croft

14 Equation of creatinine clearance in child?


Schwartz equation

15 procss of using the drug is ?


Administration

16 What is function of golgi bodies?


Energy production

17 if the mean age of participant is 50 with 5 standard deviation meaning ?


Within 5 of the age
18 Carbohydrate polymer ?
Glucagon

19 GRAIN=
.065 gram or 65mg

20 Which association that define Thalidomide disaster ?


kefauver-harris amendment

21 Which one carry high risk for IV admixture?


Preparing IV admixture from non-sterile products

22 Central tendency affected by skewing values which one?


Range

23 prescription label should contain ?


physician name

24 Unit dose cassettes in hospital has doses of ?


Day

25 Drug stored in the body in ?


fat and protein

26 The prescription of drug should contain ?


prescriber name

27 Isotonic NS and Dex solution


0.9 % saline
Or 5% Dextrose

28 which of these measures used in central tendency


Measures of Central Tendency: Mean, Median, and Mode

29 What is the best method to present Numerical data ?


In tables.

30 Concentration menthol in 0.6 syrup?


6

31 causality assessment scale


Naranjo Algorithm

32 What is benefit of OTC ?


Easley accessible for patients

33 Iso 5 particle size


no more than 3,520 par3cles equal to or larger than 0.5 microns per cubic meter of air

34 Aspirin bond ?
Coavlent
35 Role of Krebs cycle?
Energy production and biosynthesis

36 A 10 years old take vancomycin 50mg/500ml


Over dose- ADR- not enough
37 Na Nitroprusside
Slow iv infusion

38 Which one in Intravenous IV admix is ISO CLASS 5


Cleanroom

39 Fentanyl patch should avoid


Heat

40 when you say on two drugs have the same bioequivalence


Pharmacokinetic parameters
41 Normal water is used for which of these preparations?
External preparations

42 Aspartame is added to some preparation as


sweeting agent

43 The ability of liquid to dissolve in another liquid is called


Miscibility

44 liquid prep. differs from solid in


easy in administration

45 linear pharmacokinetic parameter mean


clearance don "t changed

46 method used in sterilization (autoclave was not in the choices:


dry heat

47 Obese patient what the effect of obesity on distribution of lipid soluble drug
Increase

48 Obese patient what the effect of obesity on absorption of lipid soluble drug
no effect

49 Change the drug from tablet to final powder:


Disintegration

50 LPL is responsible for transportation of:


fats to tissue

51 Most group use OTC


Adult

52 Non linear kinatic ?


Phenytoin
53 Which one of these minerals its change will increase lithium toxicity?
Na
(explanation: sodium depletion (i.e. hyponatremia) can impair lithium excretion
resulting in lithium accumulation inside the body. This’s because the kidney mistake
lithium for Na.)

54 If a drug has Mwt more than 500 in which way will excrete
Biliary

55 patient use methylprednisolone iv 20mg , he want to Change to prednisolone tablet


choose
80mg

56 Volume of blood in the body


5 liters

57 Appropriate counseling and follow-up for PQ with initiation of


Levothyroxine includes all of the following, EXCEPT:
take levothyroxine on a full stomach for greater absorption.

58 Which of the following parameters is the most appropriate for PQ's


Self-evaluation of the effectiveness of levothyroxine therapy?
Increased energy

59 T 1/2 ... in first line is ....


0.693 k

60 PH measures:
Acidity

61 If a drug is eliminated by first order elimination, what determines


when will it reach steady state?
t (1/2) half life

62 Which can be problematic with iron supplements in the elderly


Absorption

63 Drug stored in the body in ?


fat and protein

64 The prescription of drug should contain ?


prescriber name

65 drops teaspoonful
1 teaspoonful equal to 5ml
100 equal to 5ml

66 Plasma conc. Of drug in all body is ?


rate of distribution
67 Normal range of Cr. clearance
Normal creatinine clearance is 88–128 mL/min for healthy women and 97–137 mL/min for
healthy men

68 Resting membrane potential ion distribution


2Na and 3k inside while 3Na and 2k outside the inside is negative and the outside positive
the resting membrane potential is -70

69 Enzyme metabolize starch


a-amylase

70 Tyramine containing food interact with


Isoniazid

71 Sevelamer phosphate binder


Sevelamer, a polymeric compound, acts by binding to phosphate molecules in the gut,
limiting its absorption and thus lowering serum phosphate levels w/o altering Ca, Al, or
bicarbonate levels

72 Abbreviation lead to drug error

73 PKa for normal water in room temperature ?


14

74 about steady state concentration during IV administration ?


directly proportional to infusion rate
inversely proportional to total body clearance

75 In which place produces spinal fluid


choroid plexus

76 Boric acid in preparation


puffer

78 Buffer system in human body


Bicarbonate

79 to compare between 2 drugs use ?


Pharmacokinetic parameters effect & Pharmacodynamic parameter

80 Meaning of OMM abbreviation


Optimal medical management medicine

81 In parenteral nutrition what take in consideration when calculating


Propofol

82 Open angle glaucoma, when take his drops

83 How we measuring Synthetic liver function


Albumin
84 causes of hypo thyrodism ?
iodine deficiency , low hypothalamus &pituitary hormons

85 Role of lactulose in hepatic encephalopathy


Decrease protein absorption - Acidifier of colon to prevent ammonia absorption- Alkylation
of colon to prevent ammonia absorption

86 Naranjo scale was 9


Definitely

87 Example of 2 different drugs, similar letters, How we avoid errors when we inter the
prescription to the system?
tall man letter

88 not from WBC ?


Reticleucyte

89 The organ is responsible for energy production in the body?


Liver

90 Cush syndrome
Hyper adrenal

91 If drug in the marketing 2009 when expire ?

92 The valid of tramadol prescription


1 month

93 The most common disintergrator in compressed tablets is ?


Starch

94 Posterior Pituitary gland release hormone


Oxytocin and antidiuretic hormone

95 Cockcroft-Gault used to calculate :


Stable chronic renal impaired

96 Intangible cost?
Pain, suffering, anxiety and fatigue

97 Gums are used in tabletting primarily as ?


Binding agents

98 Latin expression means “ after meals” :


P.C

99 Reabsorption of Amino acid in renal tubules


proximal convolutions

100 Colon absorptive surface is important for


water, K
101 Normal Cr.cl
Creatinine clearance in a healthy young person is about 95 milliliters per minute for
women/120 milliliters per minute for men. This means that each minute, that person's
kidneys clear 95-120 mL of blood free of creatinine.

102 Precursor of bile


Cholesterol

103 Function of GOLGI bodies in the cell


transporting, modifying, and packaging proteins and lipids into vesicles for delivery to targeted
destinations

104 Function of mitochondria in the cell


Energy
105 Solid dosage forms are better than Solution dosage form because ?
Accurate dose stable & Easy to handle & More

106 lab test cab be done now ?


Iron

107 what lab tests have to done today:


CBC

108 total body clearance=


CLhepatic+ CLpulmonary +CLrenal

109 The latin abbreviation for(state) ?


Immediately

110 Drug after metabolized in liver it will become ?


Polar

111 PPM mean:-


part per million OR million part

112 Which of the following is colloid solution ?


Albumin

113 prn meaning ?


As needed

114 Which of the following used to counteract toxicity of a drug after 1 hour of ingestion ?
activated charcoal

115 disintegration ?!
change the drug from tablet to final powder
116 Naranjo score is 3 for propability so it is?
Naranjo score is from -4 to +13
1 to 4 possible
5 to 8 propable
9 and more definite
0 and less doubtful

117 P.C:
after meal
A.c: before meal
P.R.N: as needed
QID=QDS=4times daily

118 At 25 C, benzoic acid is much more soluble in benzene (C6H6) than it is in water. In this situation
benzoic acid could be considered to be ?
Non-polar

119 The following liquids are least likely to be miscible ?


non-polar + polar
120 enteroHepatic recycling depend on all of the following except ?
extent of drug absorption from stomach

121 arranging the electro magnetic waves alpha,, beta,„gamma in an increase ranking in air will be ?
alpha,beta.gamma

122 Hypertonic solutions can be adjusted by ?


Make dilution by add more solvent

123 Very small molecule(1 part ) soluble in more than 10,000 part of water?
Insoluble

124 The following properties are similar for both solutions and suspensions ?
The components of both solutions and suspensions can be separated from each other by physical
processes & Solutions and suspensions are composed of two or more components

125 All of the following to decrease the exposure to the radiation except ?
decrease the activity of radiation person

126 protect from light" in usp means ?


Amber glass
127 drugs have different crystal structures so different in ?
Polymorphism

128 suspension eye drop differ than ophthalmic solution in ?


less commonly used & used for insoluble drugs

129 cushing - like syndrome is due to ?


adrenal hyperplasia

130 which is (are) electromagnetic waves??


X-ray & Gama- ray

131 enzyme kinetics law ?


Michaelis-Menten law

132 most described dosage solid form ?


Tablet

133 Controlled released drug delivery depends on ?


Ready programmed&no influece of the body fluid

134 Amount of water in adult male ?


60%
male 60 % female about 55%

135 Hormone released from posterior gland ?


vasopressin and oxytocin

136 metabolism in intestine ?


Hydrolysis

137 a drug increases the effect of highly bound to plasma protein ?


Digoxin

138 when FDA needs more experiments, analysis and researches about a drug in phase 4 it
means ?
need more researches about safety

139 According to FDA the most dangerous abbreviation that shouldn't be written on
prescription is???
Prn

140 85 years man have pain in joint . what the most dangours abbreviation put in prescription ?
OD

141 hormone released from adrenal cortex Adrenocorticotropic ?


Progesterone

142 liquid dosage form differ from solid in ?


ease to administation
143 The method most commonly used in the sterilization in our society ?
Autoclave

144 in ovulation phase, Which hormone predominate?


LH

145 the LABEL of drug should has ?


Patient name

146 All of the following to decrease exposure to radiation except ?


decrease the activity of radiation person

147 One of the following is a component of drug processes ?


Administration

148 Why creatinine is used as an indicator for renal function ?


it's bound to protein that is only excreted by kidney

149 Linear pharmacokinetics ?


First order
Non - linear ? Zero order

150 The following is related to dependence ?


increasing the dose of drug associated with withdrawal symptoms if sudden decrease

151 Cockcroft - Gault Equation is used for glomerular filtration and is indicted for ?
chronic renal failure OR Bilateral renal failure & Bilateral chronic

152 DRUG Clearance means ?


The elimination of drug from the body

153 drugs that physician should care when using it ?


low therapeutic index

154 PQ is a 75 year old patient who has just been diagnosed with
hypothyroidism. Her past medical history is significant for congestive
heart failure, type 2 diabetes mellitus, osteoporosis and chronic stable
angina, all of which are well-controlled. Her medications include:

Metoprolol 25 mg bid
Calcium carbonate 1250 mg bid
Vitamin D 1000 IU daily
Glyburide 2.5 mg bid
Enalapril 10 mg bid
Furosemide 40 mg daily
Nitroglycerin SL spray prn
* PQ should be started on a low dose of levothyroxine because of her

age.
155 Define maslow needs in order

156 Which type of medication that can be crushed?


Immediate release tablet

157 Non linear kinatic ?


Phenytoin
Structure

Benzodiazepines

Barbiturates

Quinidine

Convert codeine to morphine by ( give structure of two drug)

demethylation
(aspirin structure) then ask which SAR related to its antiplatelet activity

Which one is double bonds structure ?

Structure of quinine?

Methacholine structure

dihydropyridine structure ?
Structere of Oseltamvire

structure of phospholipids
hydrophilic head and hydrophibic tail
Definitions

1 What is Drug dependency


Dependence develops when the neurons adapt to the repeated drug exposure and only
function normally in the presence of the drug

2 Management definition
Management is the process of dealing with or controlling things or people. (In general)
Management is both an art and science. As a science, it seeks to know variables that affect
performance and as an art, it uses the function of coordinating to determine the
performance of workers.

3 Pharmacophore definition?
It is part of molecule bind to receptor responsible for activity

4 A person did something good the behavier is called


Benifience

5 Definition of bioavailability
Fraction of a dose of drug that is absorbed from its site of administration and reaches, in
an unchanged form, the systemic circulation

6 chemotaxis
Movement of a motile cell or organism, or part of one, in a direction corresponding to a
gradient of increasing or decreasing concentration of a particular substance

7 p value
The p-value is the level of marginal significance within a statistical hypothesis test
representing the probability of the occurrence of a given event.

8 Definition of Strategy plan


A systematic process of envisioning a desired future, and translating this vision into
broadly defined goals or objectives and a sequence of steps to achieve them.

9 What is meaning polypharmacy ?


Polypharmacy is defined as the simultaneous use of multiple drugs to treat a single ailment
or condition, or the simultaneous use of multiple drugs by a single patient, for one or more
conditions. As the population is aging, polypharmacy has become an important risk factor
for poor outcomes in the elderly

10 What is meaning pharmacovigilance?


the study of the safety of marketed drugs under the practical conditions of clinical use in
large communities – involves the paradox that what is probably the most highly regulated
industry in the world is, from time to time, forced to remove approved and licensed
products from the market because of clinical toxicity.

11 Polymorphism definition

12 Private between you and patient definition


Confidentiality

13 Autonomy
Requires that the patient have autonomy of thought, intention, and action when making
decisions regarding health care procedures.
14 Justice
The idea that the burdens and benefits of new or experimental treatments must be
distributed equally among all groups in society.

15 Beneficence
Requires that the procedure be provided with the intent of doing good for the patient
Involved

16 Non-maleficence
Requires that a procedure does not harm the patient involved or others in society.

17 Phentolamine
alpha blocker

18 what is mean (anergy) in immunity


lack of reaction by the body's defense mechanisms to foreign substances

19 toxidrome ?
over poison induced syndrome

20 tolerence ?
patient need more dose to obtain the same effect

21 Following the ethical principle of nonmaleficence requires that pharmacists ?


do good to patients, placing the benefit of the patient above all else.

22 Tolerance ?
increasing dose of drug to obtain the same effect

23 High alert drugs


small group of medications in any health organization that medical staff deal with them in caution
and their loss from the place cause a very serious problem so there is special way to store and
dispense them

24 bureaucratic
The style of management in which the owner of a pharmacy emphasizes the development of
detailed policies and written procedures for employees to observe at all times .

25 The ethical principle of veracity requires that ?


we act with honesty, without deception

26 - If a drug has the same active ingredient like other drug but not contain the same inactive
ingredient this mean ?
pharmaceutical equivalent

27 Antidote :
antagonism of the toxicity of over dose of a drug

28 tolerance ?
increasing dose of drug to obtain the same effect
29 dependence ?
increasing the dose of drug associated with withdrawal symptoms if sudden decrease

30 drug dependence means ?


Addiction

31 another definition for a soluble solution ?


Homogenous

32 FDA Fast Track what this mean ?


drug that shows promising results for life-threatening disease with NO others available can do that

33 which of the following used to deal with adverse drug effects ?


Pharmacovigilance

34 half-life(t1/2) ?
Time it takes for the plasma concentration or the amount of drug in the body to be reduced by 50%

35 First-Pass Effect
Blood that peruses virtually all the gastrointestinal tissues passes through the liver by means of the
hepatic portal vein

36 toxiderm meaning :
the syndromes caused by toxin

37 Bioavailbility
Area under the curve

38 Toxidrome
physical signs and symptoms due to specific toxicity syndrome

39 Compassionate drug
Compassionate drug use (or sometimes just compassionate use) is the use of a new, unapproved
drug to treat a seriously ill patients when no other treatments are available

40 Pharmacovigilance
the practice of monitoring the effects and safety of medical drugs aier they have been licensed for
use, especially in order to identify and evaluate previously unreported adverse reactions.

41 What is the Planning definition ?


*Strategic planning:
provides a framework for more detailed planning and day to day decisions.

*Operational planning :
Prepared perform the immediate tasks

*Business Planning:
To determine the feasibility

*Resources planning:
The resources necessary to achieve the goals and strategy of the organization.
*Organizational planning:
Challenging of the futures

*Contingency planning:
To provide a fallback option

42 Selection Bias?
selection bias : the study in cleveland is not representative of the US
This type of error occurs when a systematic error in the enrollment of individuals in a study causes a
biased estimation of the association between exposure and outcome

43 Pharmacoveglince :
science and activities relating to the detection, assessment, understanding and prevention of
adverse effects or any other drug-related problem

44 Orphan drug
treat medical conditions which, because they are so rare, would not be profitable to
produce without government assistance. The conditions are referred to as orphan disease
defined as those intended for the treatment, prevention or diagnosis of a rare disease or
,condition

45 definition of relaiblity?

46 Definition of Autonomy

47 Cost benefit
pharmacoeconomic study to compare two drugs based on monetary ( dollar)

48 Cost minimization
pharmacoeconomic study to compare two equivalent drug and choose the one with lowest cost .
Diabetic

1 Insulin last 24 hrs one dose a day?


Glargine

2 Plate diet used for


Diabetic

3 Insulin take once daily ?


Glargine

4 What is the timeline in which symptoms of type 1 diabetes occur?


75% b cells destruction

5 Which metal is needed for insulin function?


Cr

6 Insulin used as IV
Regular

7 Drugs that causes hypoglycemia mask


Beta blockers

8 Insulin use in long time


Glargine

9 Side effect of metformin


Lactic acidosis

10 Patient develop side effect with metformin


Take with with meal

11 Best place to inject insulin


Abdomen

12 Patient has diabetics what drug is contraindicated


Beta blockers because it mask hypoglycemia

13 Drug mask hypoglycemia


Carvedilol

14 Side effect of oral hypoglycemic agent


Hypoglycemia

15 An 18-year-old boy was diagnosed with diabetes type 1 the doctor wanted to give him
poles/basic insulin thereby to reach normal glycemic control ?
30 mg insulin glargine once a day

16 Drug mask hypoglycemia


B-blockers

17 Drug for diabetics need adjustment


Alogliptin, saxagliptin, and sitagliptin are orally active dipeptidyl peptidase-4 that need
dose adjustment
18 When patient take Metformin
With meal as divide dose

19 Metformin of label use


Ovarian cystic in woman

20 High risk of hypoglycemia


Insulin

21 When DM type 1, symptoms appears


70% destroy of Langerhans cell

22 patient A1C >= 9% consider start?


dual therapy

23 patient 48 years old male known case of hypertension, newly diagnosis diabetic type 2,
A1C 6.8%, what the best option to start treatment of patient?
Life style modification + Metformin

24 Metformin classification and contraindication


Metformin [met-FOR-min], the only biguanide, is classified as an insulin sensitizer

25 patient is 16years old female, Wt:55kg and Ht: 158, patient known case of diabetic type 1
How much unite “basal insulin glargine (lantus)” patient takes?
Basal insulin is started at 0.1-0.2 units/kg/days
6 units-11 units

26 what type of oral hypoglycemic agents can’t use with insulin? Why?
Meglitinides and sulfonylureas
Risk of hypoglycemia

27 as pharmacist and patient came with prescription for DM Containing ( metformin,


glimepiride, thyroxine, Omeprazole, aspirin and atorvastatin) patient has sulfa allergy,
what you do?
Glimepiride is sulfonylureas class, contraindication with sulfa allergy, type 1 diabetes, DKA
So D/C Glimepiride and referral the patient to doctor with recommendation

28 which Oral hypoglycemic agent contraindication with heart failure?


Pioglitazone is Thiazolidinediones is black boxed warnings cause or exacerbate heart
failure
And contraindication class III/IV heart failure

29 which Oral hypoglycemic causes Acute pancreatitis?


Dipeptidyl peptidase 4 (DPP-4) inhibitors Such as: sitagliptin

30 what first line of treatment hypoglycemia?


Glucose is preferred treatment

31 tyrosine kinas class II receptors are ?


insulin receptors
32 desmopressin used in treatment of ?
nocturnal enuresis and diabetes insipidus

33 blood glucose of patient 210 mg/dl and target 120, the correction factor is 30. What the
correction dose for this patient
Correction dose= (blood glucose now-target blood glucose) / correction factors
(210-120) /30
=90/30
=3 units
Add to patient dose

34 Off label use metformin


polycystic ovary syndrome , weight lose

35 patient was in C.T with contrast procedure what is the drug should to be stopped for the
next 48 hour
Metformin

36 Drug need renal adjustment


Sitagliptin

37 oral diabetic used for type 1


( SGLT2 inhibitors)

38 drug avoided in acute kidney insufficiency


Metformin

39 long acting) insulin is ?


Glargine

40 insulin given iv is ?
Regular

41 metformin and metronidazol cause ?


metallic taste

42 What first line agent may be considered for an obese type 2 ?


Metformin

43 which of the following diabetic drugs approved by fda for pediatric use ?
Metformin

44 drug of choice for gestational diabetes ?


Insulin

45 anti diabetic drug used pregnancy?


Metformin

46 Drug exaggrated blood glucose ?


Hydrochlorothiazide
47 Drug exaggrated insulin ?
metoprolol(masking hypoglycemic symptoms)

48 which of the following cause metallic taste ?


Metformin

49 A diabetic old patient with hyperlipidemia complains of erectile dysfunction so DOC ?


Sildenafil

50 12 years old boy has diabetes type1 which drug can take?
Metformin

51 Treatment of Diabetic patient with Glycated Hb1 9 as initial ttt ?


Metformin

52 patient take 4 drugs . he did kidney function test and the result was high serum cr and high BUN (
blood urea nitrogen)and high potassium serum level so which drug should be stopped ?
Metformin

53 22 years woman wants to become a pregnant, she is taking metformin and pioglitazone. she has a
history of hypoglycemia and she prefers oral therapy . What will be the best approach in her case?:
stop pioglitazone and titrate metformin

54 An elderly man around 60 years old , complain from polyuria , dry mouth . there is no family history
of diabetes he has done a lab tests and the results were positive for diabetes . initial treatment
should be ?
Metformin

55 CASE WOMAN Hyprttention diabetic and sensitive of sulph. which is contraindicated with sulpha ?
Glyburide

56 Pioglitazon side effect ?


Hypoglycemia

57 on treatment of insulin, which should be monitored


Potassium as it cause hypokalemia

58 A case of 22 years old woman that wants to get pregnant, she takes Pioglitazone with metformin as
her treatment. She has a history of hypoglycemia and she prefers to take oral therapy. What will be
the best approach in her case?
stop metformin and titrat the pioglitazon

59 case in which glucose is high and its weight 103 kg and ha1c is 9, he will take .
Metformin

60 Which type of insulin that have very long duration of action ?


Glargine 12 hour
Detemir 12hour
Degludic insulin 24hr

61 Metformin off label use


weight loss
62 G6p enzyme activated when
Incrased insulin

63 Long basal insulin ?


Glargine

64 What s the treatment of Diabetic insipidus?


Desmopresson

65 First line in treatment of diabetic patient ?


Merormin

66 Case Patient take metformin and A1c = 6.8 , what is your recommendation ?
no change on metformin
Child

1 In child with ADHD what is the appropriate choice?


Methylphenidate once daily

2 Methylphenidate indication
Attention deficit hyperactivity disorder (ADHD)

3 Neonates term
Suggested Age Groupings:
Premature Newborns: < 38 weeks’ gestational age
Term Newborns: > 38 weeks’ gestational age
Neonate: 0 – 30 days of age
Infant: 1 month – 2 years
Young Child: 2 – 6 years
Child: 6 – 12 years
Adolescent: 12 – 18 years

4 Which drug contraindicated in neonate?


Smx-tmp

5 Neonate doctor prescribe to him erythromycin eye ointment for ?


Chlamydia

6 Drug contraindication in children less than 12 year ?


At age
1- 28 days – ceftriaxone –
< 2 yeas – promethazine – fatal respiratory depression
< 12 yeas – codeine and tramadol – lethal overdose

7 Dose of dry cough syrup for child


Dextromethorphan
Cough suppressant
Adult: As dextromethorphan hydrobromide: As conventional preparation: 10-20 mg 4
hourly, or 30 mg 6-8 hourly. As extended-release preparation: 60 mg 12 hourly. Max: 120
mg daily. As loz: Suck not more than 12 loz daily.
Child: >12 years Same as adult dose.

8 Neonate with chalamedia infection what is the best antibiotics


Erythromycin

9 Drug contraindicated in neonates


Trimethoprim/sulfamethoxazol

10 Neonates water soluble ?


80%
11 Medication cause growth retardation
Glucocorticoids
Betamethasone
Cortisone
Dexamethasone
Hydrocortisone
Methylprednisolone
Prednisolone
Prednisone
Triamcinolone

12 Drug has role in RSV ( respiratory syncytial virus ) in newborn ?


Palivizumab for prevention and treatmen

13 Case children with fever 48hr, tugging ear diagnosis


otitis media

14 Most OTC for children


Antipyretic / 2-Analgesic

15 Which drug contraindicated in children


Doxycycline

16 Child has penicillin resistant which drug will be used


Cephalosporin

17 Child 4 years old has non-productive cough


Dextromethorphan

18 Child patient with ADHD, symptoms appear in house and in school


Methylphenidate

19 Ceftriaxone what cause on neonate


Ceftriaxone cause hyperalbunimia in neonate

20 gray baby syndrome caused by ?


Chloramphenicol

21 for otitis media in children use ?


high dose of amoxicillin

22 for a 4 year old child, the maximum daily dose of paracetamol is ?


1g

23 A Child with otitis media ?


high dose amoxicillin
GIT

1 In Irritable bowel syndrome what is the most causative factor?


Psychosocial stressor

2 what is equivalent to omeprazole 20 mg?


Pantoprazole 40 mg

3 Bromocriptin which medication to prevent nausea should be given


Domperidone

4 Action of secretin hormone?


Regulate secretions in the stomach, pancreas and liver.

5 Which nsaids is associated with less gastric side effect (structures)


Ibuprofen

6 Patient taking omeprazole and still suffering from heartburn?


Add Histamine antagonist

7 Cetirizine which class


H1

8 Which the following treat gastric disease


H2

9 Acute crohn disease


IV hydrocortisone

10 Which assist liver function


Albumin

11 Treatment of crohn disease


Adalimumab, Infliximab, Methotrexate
Beneficial in the treatment of Crohn disease.
6-Mercaptopurine

12 phase 2 metabolism
Conjugation

13 Which is most likely to cause heartburn?


Kcl

14 Domperidone is use in?


Nausea

15 Diarrhea treatment
Electrolytes and fluid or give antimotilty agent
16 Maximum Duration of Domperidone
7 days

17 Woman take Rivastigmine should take for nausea


Domperidone

18 Case lab result all normal except Mg. which drug cause this
Gaviscon

19 Esomeprazole for Esophageal injury ?


30 day

20 Drug cause more peptic ulcer


Ketorolac

21 Case of patient e GERD make lab data and results as following (Ca . Na .K. hco3) etc. were
in normal range) Abnormalities in test results is due to
Cimetidine

22 The following data is wrong about domperidone except


Anti-vomiting without CNS effect

24 Drug used in treatment of traveler diarrhea?


Bismuth

25 Drug cause GIT side effect


Naproxen

26 women take drug for GI ulcer and her lab data shows hypomagnesemia which drug is
responsible for that?
Omeprazole

27 Drugs that cause constipation

28 GIT ulcer in elderly

29 Patient do gastrectomy Know his Wight 55 kg Past 6 months was 85 kg Type of


nutrition state
At high risk of malnutrition

30 Which one of Osteoporosis drug can cause esophagus’s ulcer


Alendronate

31 Omeprazole advice
Every day, at morning before 30 -60 min of breakfast- Every day after 30-60 min- Any time

32 for long time he using NSAID that cause ulcer


prostaglandine analuge

33 prostaglandine analuge
stay in bathroom for 30 min in the morning - be hydrated all day
34 celiac disease
Gluten free diet

35 The prophylaxis in case of traveler diarrhea?


bismuth subsalicylate (BSS)

36 Mild Charon’s inflammatory bowel disease treatment by


Meclizine melamine

37 Esophageal candidates
Caused by c.albicans treatment by flucnazol

38 20 mg omeprazole
300 mg cimetidine

39 Gallstones called
Cholethiasis

40 The drug is absorbed from the GI tract and passes via the portal vein into the liver where
some drugs are metabolized
first pass effect

41 patient with ulcerative colitis TTT on cortisone and mesalazin must use what ?
Calcium

42 which drug will be more absorbed in presence of ranitidine ?


Naproxen

43 the drug that most likely causes peptic ulcer ?


Indomethacin

44 Sodium bicarbonate antacid makes an out elimination of ?


pka=1.2, weak acid

45 which of the following can neutralize acidity and treats gastritis resulted from NSAIDS ?
proton pump inhibitors

46 Which of the following dilate the lateral esophageal sphincter ?


Amlodipine

47 The most rapid relief of acidity ?


antacid (Al/Mg hydroxide)

48 all these drugs are anti metabolite execept ?


Cisplatin

49 which of the following decrease gastric emptying rate?


atropine and hypothyroidism

50 The following statements are wrong about domperidone except ?


anti-emetic without CNS effect
51 which of the following is an antimetabolite and anti-cancer ?
Fluorouracil

52 Omeprazole can be used as a single drug in ?


gastritis as a side effect of Nsaids

53 drug for esophageal reflux ?


Pantoprazole

54 perphenazin used in (anti psychotic)?


nausea and vomiting

55 the oesophageal ulcer occurred due to ?


increase gastric secretion

56 cyclizine is used in ?
nausea and vomiting

57 Which organism is the cause of travelers’ diarrhea ?


E.coli

58 omeprazole used ?
before 30 minutes breakfast

59 antacid is used in heart burn to ?


neutralize acidity

60 magnesium sulphate( antiacid) has ?


cathartic side effect so used as laxative

61 treatment of h.pylori infection by triple therapy ?


(proton pump inhibitor+2 antibiotics) (omeprazole+clarithromycin+(amoxicillin or metronidazole) )

62 diphenoxylate and loperamide are narcotics or opioid agonists which are used as ?
anti diarrheal

63 when we need to use omeprazole as a single therapy?


Gastritis

64 which drug will be more absorbed in presence of rantidine ?


Naproxen

65 Patients with ulcerative colitis ttt on cortisone and mesalazin must use what?
if Colitis & Cortison ... .>> the answer is Ca
if Colitis only. .>> the answer is vit b12

66 DRUG cause more peptic ulcer:


Indomethacin

67 Which of the following is given to neutralize stomach acidity and prevent peptic ulcer
Antacid
68 drug-drug interaction with plavex..
Omperazol

69 anti acid make complex with:


doxycycline

70 most low potency corticosteroid


Hydrocortisone

71 which drug has the rapid effective action in esophageal burn and acidity
magnesium / aluminum hydroxide

72 Omeprazole alone is used for ttt of


Gastritis

73 which antacid give cathartic effect as a side effect


Mg hydroxide

74 A patient takes clopidogrel and omeprazole so should shift to?


Pantoprazole

75 which is most likely to cause heartburn


KCL

76 PPI (Proton Pump Inhibitor) is used in GIT bleeding is:


Pantoprazole

77 What is the equivalent dose for Esomeprazole 20 mg?

78 Travelar diarrhea prophylaxis?


Hygiene

79 Case patient exposed to cow and have symptoms what should give ?
azithromycin to prevent risk of brucellusis

80 Case old man with UTI what should give prophylaxis ?


Nitrofurantoin

81 Which is the best alternative if patient with H.Pylori previously treated with triple therapy and
resistance to ampicillin ?
Use Quadruple therapy

82 Infant take 100 tablet multi vit containing 18mg iron and came to hospital Doctors should do
gastric lavage#
activated charcoal#
activated charcoal multi doses#
charcol more safe in infant and can give multi dose
Activated charcol give during one hour

83 What is recommended for Celiac syndrome patient ?


gluten free diet
84 Drug induce pneumonia with chronic use?
PPI

85 the cornerstone in treatment of diarrhea ?


Fluid and electrolyte management

86 first line in treatment of NSAID induced ulcer ?


PPIS
Vaccines

1 Vaccine for oral use?


Rotavirus, adenovirus, cholera vaccine, and oral typhoid vaccines

2 Vaccines in pregnancy?
Tdap vaccine and flu shot (inactivated live influenza)

3 Tuberculosis vaccine in baby?


BCG vaccine at birth

4 Vaccine in hajj?
Meningococcal vaccine

5 Vaccination for older patients


If ≥ 50 years’ shingles
If ≥ 60 yeas pneumococcal disease vaccine

6 Patient had accident with deep wound , unknown history of vaccines


Tetanus (Td) and tetanus immunoglobulin

7 Case of deep wound and history of vaccination unkown ?


Give Td vaccine and TIG immunoglobulin

8 Which one has live vaccination differed ?


Child just ended course of chemotherapy

9 Which one has live vaccination differed ?


Pregnant woman

10 Which one of these vaccines has the max age when it received?
Pneumococcal

11 Which one of these has vaccination?


Varicella

12 Vaccination of tuberculosis when?


At birth

13 Doctor prescribe infliximab immunosuppressant to rheumatoid pt and he want to take live


vaccination so what to do?
Give live vaccination 2 weeks before starting infliximab

14 At which time quadrivalent meningococcal vaccine mcp4 is given in Saudi?


9 months

15 Vaccine for DM?


influenza vaccine , Pneumococcal vaccines

16 Which vaccine given to neonate during 12 hrs of birth?


Hepatitis B

17 Vaccination at birth
BCG and Hepatitis B
18 Human piploma Vaccine use in?
Cervical cancer

19 MMR take in?


12 month and 4 years

20 Non deep wound


Td only

21 Who is responsible for vaccination during haj


MOH

22 Vaccine TB any age


Birth

23 Active and not active vaccine what you do


Live and inactivated vaccines (combo) can be administered at the same
Time or without regards to the spacing

24 Human papilloma virus for ?


Cervical cancer

25 BCG vaccination is given


At birth

26 Rota vaccine CI in patient allergy to


Egg

27 nurse get infected with hepatitis B by an injection from a patient and she never took hep
B vaccination before
Take vaccine hepatitis and immunology in the deferent site

28 Vaccine in Pregnancy
Tdap

29 IPV allergy
Neomycin

30 Varicella
Another name for chickenpox it is caused by the varicella zoster virus

31 Vaccine taken once


BCG

32 Which vaccine is oral


OPV, RV are oral vaccines

33 When Influenza vaccine should be taken


October to march

34 Pilgrims vaccine is
meningococcal
35 immunocompromised child take I.V I.G and time for receive MMR vaccine is now ..what
is the best action to do :
sould not take vaccine any time

36 A patient took immunoglobulin how long you should wait before administering vaccine
2 weeks

37 which of the following vaccines should be administered for every pregnancy?


Tdap

38 routine medication for influenza is ?


influenza vaccine

39 emergency members should take the following vaccine to avoid noscomial infection ?
meningitis

40 influenza vaccine contraindicated ?


in (0-6) months baby

41 live attenuated vaccine ?


measless,mumbs & rubella (MMR)

42 Which of the following vaccine should be taken although you don’t need it ?
influenza vaccine

43 influenza vaccine is ?
safe for pregnant

44 which vaccine should be taken every year ?


influenza vaccine

45 Emergency staff's vaccine ?


Meningitis

46 A Child born to a hepatitis B positive mother must take ?


first dose of the hepatitis B vaccine and one dose of the Hepatitis B Immune

47 One of the following is contraindication to all vaccines ?


CD <200

48 Pregnant woman have hepatitis B when deliver we must give for baby?
Hepatitis B vaccine with immunoglobulin hepatitis.

49 drug used for prevention of influenza ?


Vaccination

50 Don't give vaccines to ?


Patients with CD4 < 200

51 2 years old child come to clinic for taking hepatitis vaccine , we know that he took pneumonia
vaccine from month ago , so we should ?
give him the vaccine immediately
52 Thimerosal (mercury)containing vaccine may cause ?
Autism

53 Cervical cancer caused by or treatment of infertility or to prevent cervical cancer ?


HPV (human papilloma virus vaccine)

54 What is the vaccine that reduces the incidence of infertility or prevent infertility ?
human papilloma virus

55 a nurse was giving tratment to a hepatitisB patient when she infected with his blood, when she
made analysis the result was HBSAG negative and HBSAB negative, so she should be treated
with ?
give her immunoglublin + hepatitis B vaccine

56 Influenza vaccine in pregnancy is?


Safe (IV) Inactivated influenza

57 What is the recommended vaccine in patient with Deep wound and unknown history of vaccines?
TIG +Td
Deep wound + Unknown Vaccination history = Td + TIG
Deep wound + Patient has vaccinated within 5 years = No vaccine needed
Deep wound + patient are not vaccinated within 5 years = Only Td
Minor wound + Unknown vaccination history = only Td
Minor wound + patient are vaccinated within 10 years = No vaccine needed
Minor wound + patient are not vaccinated within 10 years = only Td

58 Which Vaccine stored in freezer?


Zostavax , MMR can be store both in freezer and refrigerator

59 which of the following prevent the cervical cancer in women?


Human papillomavirus (HPV) vaccination

60 What type of vaccine that given to pregnant women ?


Tdap

61 At which age the Meningitis vaccine should given ?


Two doses ( at 18 month , 24 month )

62 Which of the following are considered live vaccine ?

63 Baby with immune problem , When he must take the MMR vaccine ?
He will take MMR on delay

64 Which vaccine that given in pilgrims ?


Annual influenza (flu) vaccination ( before 2 weeks) If not already administered
MCV (Meningococcal ACWY) (every 3-5 years)
65 Which vaccine that given in single dose?
BCG vaccine given once at birth

66 When to give MMR vaccine in Saudi Arabia ?


3 month

67 vaccine for new born in saudi arabia ?


HIB B and BCG

68 vaccine for pregnant with HIB B what we give the baby ?


Both Vaccine and immunoglobulin
Genomic

1 CYP2D6 with codeine when the effect is reduce?


Poor metabolizer

2 Allopurinol make
severe cutaneous adverse reactions (SCAR)
positive for HLA-B*58:01 allele

3 Which is purine in DNA?


purine [ A,G]

4 DNA?
deoxyribonucleic acid

5 Trastuzumab action on breast cencer on which?


HERE2

6 Codeine CYTP2D6 poor metabolizer?


suffer more pain

7 Which one associated with genetic polymorphism?


Warfarin

8 Which one is tumor suppression protein?


P53
9 Question about KRAS and HER2 and what is the treatment?

KRAS
KRAS mutations are associated with CRC prognosis-
- Determine mutation status with tumor KRAS genotyping at diagnosis. Epidermal growth
factor receptor (EGFR) inhibitors should be considered only in patients with tumors with
wild-type KRAS.
EGFR-Inhibitors(Cetuximab, Panitumumab)
Chemotherapeutic Regimens
-FOLFOX plus panitumumab
-FOLFIRI plus cetuximab
-FOLFIRI plus panitumumab
-Cetuximab plus irinotecan
-Panitumumab plus irinotecan
-FOLFOX or capox ± cetuximab or panitumumab
-capox, capecitabine plus oxaliplatin , FOLFIRI, fluorouracil plus leucovorin plus irinotecan;
FOLFOX,fluorouracil plus leucovorin plus oxaliplatin.
-Test tumors for KRAS mutations at diagnosis of stage IV disease; patients with mutations
on codons 12 and 13 on chromosome 12 are not candidates for EGFR inhibitors
HER2
-HER2/neu (HER2) overexpression is associated with transmission of growth signals that
control aspects of normal cell growth and division. Overexpression of HER2 is associated
with increased tumor aggressiveness, rates of recurrence, and mortality
-The use of trastuzumab with chemotherapy is appropriate for patients
With HER2-positive tumors.
-Three anti-HER2 agents are available: trastuzumab, lapatinib, and pertuzumab
-Pertuzumab is a monoclonal antibody that binds to a different HER2 domain as compared
with trastuzumab
10 Which antiviral c combination can be applied to all genotypes?
Sofospuvir and velpatasvir

11 Warfarin drug interaction? CYP2C9


Increased INR w/ CYP2C9 (e.g. Amiodarone, capecitabine, cotrimoxazole),

12 Clopidogrel drug interaction? CYP2C19


May reduce antiplatelet effect w/ CYP2C19 inhibitor including PPI (e.g. Esomeprazole,
omeprazole).

13 Abacavir whats gene Of it


HLB b 5701

14 What is two component of purine nitrogen base


Adenine , Guanine

15 What is the basic component of the cell


Cell membrane or plasma, cytoplasm and nucleus

16 CYP2C19
Clopidogrel

17 Patient diagnosed with HIV doctor wants to start treatment among the medication
prescribed was abacavir what is the next step should be taken
Test for HLA-B*5701

18 drug interactions between clopidogrel and omeprazole through which enzyme ?


CYP2C19

19 Pharmacogenomics of cetuximab
KRAS mutation (-ve)

20 allopurinol genetic is HALA_B will be make. side effect of


sever cutaneous

21 Carpemazine genitic HALA_B


epidemic necrosis

22 Which of the following is recombint DNA product


Livagluaide

23 Ig cannot cross the placenta


IgA, IgA,IgE .. these cannt cross placenta

24 Clopidogrel interaction omeprazole


Omeprazole and other PPI are Strong CYP2C19 inhibitors reduce antiplatelet effect

25 Abacavir gene
HLA-B 5701

26 Clozapine + smoking
Decrease clozapine plasma
27 what is cytochrome subtype responsible for drug interaction between omeprazol and clopidegrol ?
CYP2C19

28 genetic test for azathioprine


TPMT

29 cytochrome responsible for interaction between omeprazole and clopidogrel


cyp2c19

30 Abacavir test and which class?


HLAB 5701

31 Question about role of RNA ( snRNA)


CNS
1 Status epilepticus treatment
Benzodiazepines are the first-line treatment for convulsive status epilepticus because they
control seizures rapidly
For IV therapy, lorazepam is preferred in adults [6]; midazolam is preferred for
intramuscular (IM), intranasal or buccal therapy; and diazepam is preferred for rectal
administration.
Lorazepam – Use of lorazepam as a first-line agent

2 Amantadine MOA
Anti-Parkinson Agent inhibiting the N-methyl-D- aspartate (NMDA) type of glutamate receptors.

3 The dose of levodopa /carbidopa depend on which one of them?


Carbidopa

4 Mechanism of Action of chlorpromazine?


Blockade of dopamine 2 receptors (D2) is responsible for reducing the positive signs of pyschosis
& improving other behaviors.
The combined effect to block D2, histamine H1 & muscarinic M1 receptors in the vomiting center is
postulated to reduce nausea & vomiting.
Blocks α-1 & 5-HT2 > D2 >D1.

5 Isoniazid is given with pyridoxine?


10 to 50 mg daily, to reduce the incidence of central nervous system (CNS) effects or
peripheral neuropathies.

6 Mechanism of donepezil in Alzheimer


reversible acetylcholinesterase (ache) inhibitor

7 Syncope from first dose ?


Prazosin

8 Lamotrigine side effect ?


Rash

9 Which one of antiepileptic drugs need renal adjustment ?


Topiramate

10 New drug for Alzheimer will be released so MOA will be?


inhibit B amyloid peptide

11 Patient obese has depression which antidepressant is best?


Buprobion

12 First line in status epileptic?


Lorazepam

13 Drug for severe Parkinson

14 Fastest anesthetic onset and recovery


Halothane
15 In which part of the brain dose Parkinson drugs make its effect
Therapy is aimed at restoring dopamine in the basal ganglia and antagonizing the
excitatory effect of cholinergic neurons, thus reestablishing the correct
dopamine/acetylcholine
Balance.

16 What is the medication to treat neuropathic pain and depression


The ADA guidelines recommend duloxetine and pregabalin first-line. Other drugs
That can be considered include tricyclic antidepressants (TCAs), gabapentin, venlafaxine,
carbamazepine, tramadol and topical capsaicin.

17 Pro drug primidone of ?


Phenobarbital

18 Phenytoin sodium equivalent to phenytoin base?


Each 100 mg of phenytoin sodium (salt) contain 92 mg of phenytoin base

19 Doctor prescribe phenytoin 100 mg TID and then change dose after how many days need
monitoring
30 days

20 Antidepressant should be avoided in seizures patient


Bupropion

21 Anxiolytic with least sedation and withdrawal symptoms


Buspirone

22 Analgesic considered as narcotic but dispensed normally


Methadone

23 Medications are common causes of both acute and chronic cognitive impairment
anticholinergic drug

24 Case of a female patient who is hypertensive and has a history of epileptic attacks and
recently she had depression? What is the best anti-depressant for her case
Paroxitine

25 Elder patient and fat and have depression what to give


Bupropion ( cause weight loss ) .

26 Hypertension and epileptic seizures have recently had a depression what is the best antidepressant
in its case
Paroxetine

27 old patient with seizure the physician prescribes diazepam what would you choose
Alprazolam

28 Drug of antiepileptic need to adjustment


Topromycine
29 Treatment of eclampsia in seizures in woman
Magnesium sulfate

30 Side effect of benzodiazepines


Drowsiness and confusion are the most common side effects of the benzodiazepines.
Ataxia occurs at high doses Cognitive impairment

31 Antipsychotic treat the symptoms of schizophrenia

32 levodopa/carbidopa absorption affected by


empty stomach

33 psychiatric patient use olanzapine - lab result high glucose Which alternative
Haloperidol

34 What are the catecholamine


Epinephrine norepinephrine and dopamine

35 Smoking with olanzapine


This drug may cause somnolence and dizziness, if affected, avoid driving and operating
machinery. Avoid cigarette smoking

36 Clozapine monitoring
White blood cells

37 DOC absence seizures?


Ethosuximide.

38 Naloxone dose in children


As naloxone hydrochloride: 100-200 mcg, may give additional 100 mcg if necessary
at 2-3 minute intervals until adequate response is obtained
As naloxone hydrochloride: 10-20 mcg/kg at 2-3 minute intervals until adequate
response is obtained

39 Most effective medication in treating positive symptoms in schizophrenia


2nd generation

40 tyramine when taken with MAOIs (monoamine oxidase inhibitor) cause severe crysis of.?
Hypertension

41 tertiary amine of TCA (tricyclic antidepressant) is ?


Amitriptylline and imipramine
42 Benzodiazepine is used in treatment of ?
( Anxiolytic and anticonvulsant ) anxiolytic

43 Child on oxcarbazepine for epilepsy and suffers from rash ?


Change to ethosuximide

45 ethosuximide ?
anti epileptic used in absence (petit mal) seizure

46 Drug of choice in absence seizures ?


Ethosuximide

47 Antidepressant helps in stop smoking ?


Bubropion

48 thiopental is ?
short acting barbiturates

49 Which of the following side effects is associated with the use of norepinephrine ?
Ischemic damage due to vasoconstriction and tissue hypoxia.

50 Ultra-short acting barbiturates are used primarily as ?


Preanesthetic agents

51 Which of the following drugs is not a Tricyclic Antidepressant?


Maprotiline (tetracyclic)

52 which one used to trigeminal neuralgia ?


tegretol(Carbamazepine)

53 used for alzehimer ?


Donepzil

54 which of these have interinsc sympathomemtices effect ?


Pindolol

55 antipsychotic cause agranulocytosis ?


clozapine atypical

56 amantadine(antiviral) used for ?


treatment of parkinsonism

57 fluoxetine is ?
SSRI(selective serotonin reuptake inhibitors)antidepressant

58 antibiotic which cause yellow teeth tetracycline ?

59 lorazepam is ?
hypnotic at dose of 4mg
60 anti cholinergic cause ?
Constipation

61 Which of the following drugs lead to addiction ?


Benzodiazepine

62 prodrug of phenobarbitone and Used as anti epileptic


Primidone

63 14years-obese girl comes to the clinic with severe rash. She was
initiated on oxcarbazepine about 3 weeks ago for management of partial
seizures.
Her medical history is significant only for seizures. She has recently
Become sexually active and admits to inconsistent contraceptive use.
Which one of the following interventions is best for her?
Change to topiramate

64 Carpidopa act anti-parkinsonism by .


inc. dopamine

65 Benzodiazipine indication ?
Anxiolytic

67 Which antidepressant drug that used with obese patient ?


Bupropion

68 Which antidepressant drug that used with seizure?


Sertraline

69 Patient with DM had ossessive depressive disorder what use


Paroxetine

70 What is the drug of choice of obessive compulsive disorder ?


SSRI
Clomipramine or fluoxe3ne or fluvaxamaine

71 elderly using antidepressant and have high risk for fall this could linked to ?
Orthostatic hypotension
Vitamins

1 Role of vitamin C in treating cold


Decrease the duration of cold

2 Patient has red patches around mouth and fatigue, loss hair Which vitamin may deficient?
Biotin B3

3 Vitamin should be avoided in high doses in case of pregnancy?


Vitamin A

4 Function of vitamin C
Vitamin C is needed for the growth and repair of tissues in all parts of your body

5 fat soluble vitamin important for


Liver

6 Side effect of niacin


The most common side effects of niacin are an intense cutaneous flush (accompanied by
an uncomfortable feeling of warmth) and pruritus. Administration of aspirin prior to taking
niacin decreases the flush,

7 Supplement decrease risk of breast cancer


vitamin D

8 Vitamin 12 called
Cobalamin

9 Long duration of corticosteroids need


calcium + vitamin D, Vitamin C

10 vitamin fat soluble?


A,D,E,K

11 Vitamin K name ?
Vitamin K-1 Phytonadione
Vitamin K quinines

12 Patient on carbamazepine has fatigue , red sored skin around lip and hair loss this is due
to Deficiency in
Biotin

13 ascorbic acid is ?
vitamin c

14 B-carotene is precursor of ?
vitamin A(retinol

15 Daily recommended dose for vit. C in male ?


90mg/day ... if asked in female the answer is 75mg \ day

16 vit c for female ?


75mg
17 Which of the following is found in vitamin b12?
Cobalt

18 DAILY intake of vitamin A for adult female is ?


700 mcg
Adult male = 1000mcg
Adult female = 800mcg
Pregnant =900 mcg
Breastfeeding = 1200_1300 mcg

19 What is the vitamin needed with treatment by corticosteroids for 2 years for chronic disease??
Vitamin D with Ca

20 Decreased efficacy of epoietin due to deficiency of ?


B12

21 Vitamins prevent fetus abnormalities


vit b9

22 vitamine is contraindicated in pregnancy in highly doses


vit. A

23 Niacin is considered as vitamin ?


vitamin B3

24 which of the following causes scurvy?


Vitamin C

25 Other name of Vitamin B12 ?


Cyanocobalamin, cobalamin and methylcobalamin

26 Which of the following active form of Vitamin D to be used in renal failure patient ?
calcitriol (also known as 1,25-dihydroxycholecalciferol)

27 Which vitamin included in synthesis of fatty acid?


Biotin

28 vitamin that treat nausea with pregnant ?


B6

29 test for vitamin B12 ?


Schilling test
Infections

1 Athlete foot treatment?

2 Antibiotic in pregnancy?
Penicillins, including amoxicillin, ampicillin.
Cephalosporins, including cefaclor, cephalexin.
Erythromycin.
Clindamycin
3 Antiviral interact with which anti TB ?
Rifambin
4

5 If patient resistant to ampicillin?


Take ciprofloxacin

6 The recommendations and direction depend on which in substance


(trimethobrim\sulfamethoxazole)

7 Pregnant woman has candida albicans so ttt ?


Clotrimazole

8 Treatment of RSV respiratory synthial virus ?


Palivizumab

9 Patient take doxycycline tablets and adapalene gel for acne He suffer from nauses and GIT
upset what to do?
take doxycycline with plenty of water

10 Doctor prescribe high dose of tobramycin 400mg/day for resistant strain why high dose?
Because of dose dependant killing of aminoglycosides

11 Which one is administered with isoniazid in TB ?


Pyridoxine

12 Patient take adlimumab for rheumatoids and he knew it may cause malignancy what to
do?
tell him about benefits of drug versus risks

13 Antibiotic that cause hearing problem?


Aminoglycosides , Gentamycin and Vancomycin

14 Which adverse reaction is irreversible


ototoxicity of gentamycin

15 Most important Structures of viral


Nucleic acid

16 Erythromycin effect on contraception pills


No effect

17 Which cause irreversible ototoxicity and defness


Aminoglycoside
18 What is the benefit control of infection in hospitals
To protect you and others from infection

19 Dose of TMP/SMX is Depending on

20 Case has Allergy of Amoxicillin H. Pylori ?


clarithromycin +metronidazole + omeprazole

21 Lab test with high k. drug cause this


trimethoprim/sulfamethoxazol

22 Pt in home care and he has a catheter Jitter chatter Must take


Ciprofloxacin

23 Patient has pneumonia and started treatment with Azithromycin po & ceftriaxone iv after
3 day the patient is stable what is the next step
Stop azithromycin and continue ceftriaxone for 3 days

24 H. Pylori treatment in patient have penicillin resistant


clarithromycin ..ppi....metronidazole

25 Which drug make teeth discoloration


Doxycycline

26 Treatment for Candida albican?


Clotrimazole

27 TB treatment

28 Treatment for malaria?


Chloroquine

29 Passive immunization of infants against lower respiratory tract infections


Palivizumab

30 Which macrolide safe with cyclosporine in renal replacement patient

31 MRSA
Vancomycin
32 antipseudomonal
most choice cephalosporin

33 Flu treatment
Oseltamivir

34 Which one is reversible SE of aminoglycoside


Nephrotoxicity

35 Dose of ceftriaxone in meningitis


2g bid

36 Vancomycin trough peak monitoring


after 3rd dose before 4th

37 cover pseudomonas
Tobramycin

38 Pinworm treatment

39 Vancomycin cause
red man syndrome

40 MRSA drug of choice


Vancomycin is the first line in treatment of MRSA infection

41 DOC for endocarditis


penicillin G

42 Gentamicin kinetics drawn after


Right before the 4th dose

43 TMP/SMZ Side. Effect on electrolyte


Hyperkalemia may occur, especially with higher doses

44 A child has otitis media first time what is the best medication
high dose of penicillin-azithromycin- Bactrim

45 antibiotic needed in community acquired pneumonia which need hospitalization

ceftriaxone +macrolid >> hospital non ICU

Ceftriaxone plus either a respiratory fuoroquinolone or azithromycin >> ICU

46 drug need dose adjustment in renal failure


Impeniem / cilastatin

47 Pt with Low creatinine clearance - high serum creatinine kidney injury contraindicated ?
Gentamycin

48 Probenecid alters which of the following to prevent penicillin excretion?


renal tubular reabsorption
49 Drug makes complex with antacid ?
Doxycycline

50 Drugs that make complex with antacids?


(tetracyclines and fluoroquinolones)

51 drug make complex with dairy product ?


Ciprofloxacin

52 drug need to monitor its plasma level ?


Gentamycin

53 DOC tape worms ?


Niclosamide

54 which of the following is NOT macrolide antibiotic?


Chloramphenicol

55 Which one of the following cannot be dispensed without prescription ?


Doxycycline

56 which one is more effective agaienst in traveller diarrhea ?


Ciprofloxacin

57 which antibiotic more likely to cause diarrhea ?


Augmentin

58 antibiotic with diarrhea as a side effect ?


Amoxicillin combination with clavunic

59 when taking oral contraceptive with erythromycin ?


the erythromycin decrease contraceptive effect

60 drug of choice for giardiasis and amoeba ?


metronidazole

61 Which of following is the drug of choice for treatment of all forms of Schistosomiasis?
Praziquantel

62 Cephalosporin act ?
Inhibitors of cell wall synthesis

63 drug of choice for tape worms ?


Neclosamide

64 ganciclovir(antiviral) used in treatment of ?


Cytomegalovirus

65 zanamivir is used for treatment and prophylaxis of ?


Influenza
66 metronidazole used in treatment of ?
clostridium difficile

67 clindamycin cause ?
clostridium difficile

68 drug of choice for tape worms ?


praziquantel or neclosamide

69 drug need serum creatinine to be monitored ?


gentamycin (aminoglycoside)

70 acyclovir is ?
used for herpes simplex

71 drug of choice for pseudomonal aeruginosa is ?


Meropenem

72 amphotericin-b is ?
parentral anti fungal

73 rifampicin is used in ?
leprosy &tuberculosis

74 drug of choice for pneumonia is ?


Pencillins

75 which of the following is the drug of choice for giardiasis ?


metronidazole(flagyl)

76 red man syndrome caused by ?


Vancomycin

77 Penicillin's are similar in MOA AS ?


Cephalexine

78 Which of the following show penicillinase resistance


flucloxacillin (floxacillin)

79 Pregnant woman has vaginal discharge and has Candida albicans


what's the drug of choice?
Clotrimazole

80 Treatment of Pregnant with vaginal candida


Clotrimazole

81 what is the b-lactam antibiotic, its mode of action is cell wall synthesis inhibitor
Cephalosporin

82 the aminoglycoside?
Antibiotic
83 ceftriaxone
"3rd generationcephalosporin

84 ceftazidime is
third generation cephalosporin

85 Erythromycin is macrolide class act mainly


gm + ve and is safe in pregnancy

86 Maximum infusion rate for vancomycin?


5mg/ml
1ml/min
10 mg/min

87 What is the anti-fungal drug that need hepatic dose adjustments?


Caspofungin and voriconazole

88 Which Antibiotics that cover pseudomonas?


Cefepime, ceftazidime, Piperacillin, ticarcillin-clavulanate, Ciprofloxacin, levofloxacin, Amikacin,
gentamicin, tobramycin, monobactam, carbapenem

89 Which Antibiotic cover MRSA first line?


Vancomycin

90 Which Medication cause red man syndrome ?


Vancomycin

91 How could we solve the red man syndrome with vancomycin?


reduce infusion rate

92 Drug of choice for clamydia ?


Azithromycin 1 gram po ×1 or doxycycline

93 Vancomycin trough?
Vancomycin trough levels should ideally be drawn immediately before administration of the fourth
dose (within 30 minutes of the dose is acceptable)

94 How is the Coronavirus transmimed?


respiratory

95 In Agar picture which of the following sensitive ?


96 What is the treatment of ringworm and jock itches which known as tinea?
Ring worm in skin :
Ketoconazol , Clotrimazol , Miconazol , Terbinafen
Ring worn in scalp :
Fluconazol , Itraconazol , Griseofulvin , Terbinafen

97 When should vancomycin peak levels be drawn ?


Peak levels are collected 1 to 2 hours after the completion of the intravenous vancomycin dose

98 A medical student touched a patient suffering from Clostridium infection. The student use antiseptic
wipes before and after exit from patient room ?
wash with water and soap

99 Beta-lactam antibiotic is considered ?


Time dependent

100 Patient with ear Tinnitus ?


betahistine

101 When can take gentamycin sample in patient with dialysis ?


Before dialysis

102 Gentamicin peak and trough ?


trough: 30 minutes before 3rd or 4th dose.
peak: one hour after dose.

103 Lyme disease is caused by ?


Lyme disease is caused by the bacterium Borrelia burgdorferi and rarely, Borrelia mayonii. It is
transmimed to humans through the bite of infected blacklegged ticks.

104 patient with staph . aureus resistant vancomycin and all other antibiotics except linazolid , linazolid
not approved for this case what we can use ?
Use linazolid even not approved

105 which antibiotics not recommended to use due to destruction by alveolar surfactant ?
Anemia

1 What is the dose of ferrous sulfate that treat anemia?


100 TID

2 Patient has sickle cell anemia and severe pain?


hydroxyurea therapy

3 Toxicity of carbon monoxide due to?


decrease oxygen carring capacity of hemoglobin

4 First line of sepsis


Norepinephrine

5 Glucose 6 dehydrogenase deficiency (G6D ) cause


Hemolytic Anemia

6 Ferrous (Fe2+) Dosage for Anemia


325 mg BID

7 Which one is used in treatment of Methemoglobinemia


methylene blue ( The primary emergency treatment for MMGE at dose 1-2 mg/kg )

8 Dose of ferrous sulfate


325 mg BID

9 Iron advice
Empty stomach

10 Anti-malaria cause anemia


Primaquine

11 the dose of folic acid in non pregnant women is ?


400 mcg/ day
600 mcg/day >> pregnant
500>> lactating

12 erythropoiten (eprex) is used for anemia caused by ?


chronic kidney disease

13 the action of epiotin enhanced with ?


Iron

14 erythropoiten(eprex) will not be useful in case of ?


iron deficiency

15 daily dose in case of iron deficiency anemia?


1200

16 Dose in iron defecieny anaemia is ?


325 mg bid

17 Anemic patient refuses to take injection so DOC ?


Ferrous gluconate
18 Megaloblastic ?
? anemia folic acid /cyanocoblamin.......i.e (vitb9+vitb12)
Pernicious anemia. Deficiency of vit b12 .Only

19 Iron deficiency anemia treated with ?


Iron

20 Megaloblastic anemia treated with ?


folic acid and vit b12

21 Pernicous anemiea treated with ?


vit b12

22 Anemia of chronic renal failure treated with ?


epoetin

23 but if there is iron deficiency it will be treated with ?


Derbepoetin

24 daily dose of iron in iron defficency anemia is ?


100 mg 3 times daily

25 A patient with iron deficiency anemia refused to take any parenteral drugs give him
ferrous gluconate

26 A patient has megaloblastic anemia should receive?


B12 cobalt and b9 folic acid

27 Drug of choice for hemolytic anemia?


Cortisone

28 Which anesthetic drug that cause anemia?


nitrous oxide

29 drug not used with G6PD deficiency ?


Primaquine

You might also like